Mathematics Chapter 15 Probability
  • NCERT Solution For Class 10 Mathematics

    Probability Here is the CBSE Mathematics Chapter 15 for Class 10 students. Summary and detailed explanation of the lesson, including the definitions of difficult words. All of the exercises and questions and answers from the lesson's back end have been completed. NCERT Solutions for Class 10 Mathematics Probability Chapter 15 NCERT Solutions for Class 10 Mathematics Probability Chapter 15 The following is a summary in Hindi and English for the academic year 2021-2022. You can save these solutions to your computer or use the Class 10 Mathematics.

    Question 1
    CBSEENMA10009098

    A survey was conducted by a group of students as a part of their environment awareness programme, in which they collected the following data regarding the number of plants in 20 houses in a locality. Find the mean number of plants per house. 

    Number of plants

    Number of houses

    0-2

    1

    2-4

    2

    4-6

    1

    6-8

    5

    8-10

    6

    10-12

    2

    12-14

    3

    Which method did you use for finding the mean and why?

    Solution

    Number of plants

    Number of houses (fi)

    Class mark (xi)

    fixi

    0-2

    1

    1

    1

    2-4

    2

    3

    6

    4-6

    1

    5

    5

    6-8

    5

    7

    35

    8-10

    6

    9

    54

    10-12

    2

    11

    22

    12-14

    3

    13

    39

    Total

    Σfi = 20

     

    Σfixi = 162

     

    Here, we have
                             sum from blank to blank of straight f subscript straight i equals 20 comma space sum from blank to blank of straight f subscript straight i straight x subscript straight i = 162
    Now,                  top enclose straight x equals fraction numerator begin display style sum from blank to blank of end style f subscript i x subscript i over denominator begin display style sum from blank to blank of end style f subscript i end fraction equals 162 over 2 equals 8.1

    Hence, the mean number of plants per house = 8.1
    We have used the direct method for finding the mean because numerical values of xiand fi are small.

    Question 3
    CBSEENMA10009100

    The following distribution shows the daily pocket allowance of children of a locality. The mean pocket allowance is Rs. 18. Find the missing frequency 'f'.

    Daily pocket allowance (in Rs.)

    11-13

    13-15

    15-17

    17-19

    19-21

    21-23

    23-25

    No. of childrens

    7

    6

    9

    13

    f

    5

    4

     

    Solution

    Daily pocket allowance C.l.

    No of children (fi)

    Mid-value xi

    fixi

    11-13

    7

    12

    84

    13-15

    6

    14

    84

    15-17

    9

    16

    144

    17-19

    13

    18

    234

    19-21

    f

    20

    20f

    21-23

    5

    22

    110

    23-25

    4

    24

    96

     

    Σfi = 44 + f

     

    Σfixi = 752 + 20f

     

    Here, we have                 top enclose x space equals space 18 comma space sum from blank to blank of f subscript i equals 44 plus f space a n d space sum from blank to blank of f subscript i x subscript i equals 752 space plus space 20 f
    We know,                       top enclose straight x space equals space fraction numerator begin display style sum from blank to blank of end style straight f subscript straight i straight x subscript straight i over denominator begin display style stack sum straight f subscript straight i with blank below and blank on top end style end fraction
    rightwards double arrow                                18 space equals space fraction numerator 752 plus 20 straight f over denominator 44 plus straight f end fraction
    rightwards double arrow                                18 (44 + f) = 752 + 20f
    rightwards double arrow                                792 + 18f = 752 + 20f 
    rightwards double arrow                                 18f - 20f  = 752 - 792 
    rightwards double arrow                                          -2f = -40
    rightwards double arrow                                             f = 20
    Hence the Missing frequency is 20.
    Question 6
    CBSEENMA10009103

    The table below shows the daily expenditure on food of 25 household in a locality.

    Daily Exp. (in Rs.)

    100-150

    150-200

    200-250

    250-300

    300-350

    No. of house hold

    4

    5

    12

    2

    2

    Find the mean daily expenditure on food by a suitable method.

    Solution

    Here.
    We have                        sum from blank to blank of f subscript i equals 25 comma space space space sum from blank to blank of f subscript i u subscript i equals negative 7 comma space   A (assumed mean) = 225 and h = 50
    Now,                              top enclose straight x equals space A plus fraction numerator begin display style sum from blank to blank of end style f subscript i u subscript i over denominator begin display style sum from blank to blank of end style f subscript i end fraction x h equals 225 plus fraction numerator negative 7 over denominator 25 end fraction x 50 equals 225 minus 14 equals 211
    Hence, the mean daily expenses is 211.
    Question 7
    CBSEENMA10009104
    Question 8
    CBSEENMA10009105

    A class teacher has the following absentee record of 40 students of a class for the whole term. Find the mean number of days a student was absent.

    Number of days

    0-6

    6-10

    10-14

    14-20

    20-28

    28-38

    38-40

    Number of students

    11

    10

    7

    4

    4

    3

    1

    Solution

    Number of days

    Number of students (fi)

    Class mark (xi)

    fixi

    0-6

    11

    3

    33

    6-10

    10

    8

    80

    10-14

    7

    12

    84

    14-20

    4

    17

    68

    20-28

    4

    24

    96

    28-38

    3

    33

    99

    38-40

    1

    39

    39

    Total

    Σfi = 40

     

    Σfixi = 499


    Here, we have                   sum from blank to blank of f subscript i equals 40 space and italic space italic sum from blank to blank of f subscript i x subscript i italic equals italic 499
    Now,                                 top enclose straight x equals fraction numerator begin display style sum from blank to blank of end style f subscript i x subscript i over denominator begin display style sum from blank to blank of end style f subscript i end fraction equals 499 over 40 equals 12.47
    Hence, the mean number of days a student was absent is 12.48.
    Question 9
    CBSEENMA10009106

    The following table gives the literacy rate (in percentage) of 35 cities. Find the mean literacy rate.

    Literacy rate (in %)

    45-55

    55-65

    65-75

    75-85

    85-95

    Number of cities

    3

    10

    11

    8

    3

     

    Solution

    Here, we have                    sum from blank to blank of f subscript i space equals 35 comma space sum from blank to blank of f subscript i u subscript i equals negative 2 comma space h space minus space 10 space a n d space A space equals 70

    Now,                                  top enclose straight x space equals space A plus fraction numerator begin display style sum from blank to blank of end style f subscript i u subscript i over denominator begin display style sum from blank to blank of end style f subscript i end fraction cross times h
                                            
                                           equals 70 plus open parentheses fraction numerator negative 2 over denominator 35 end fraction close parentheses cross times 10 equals 70 minus 4 over 7
                                            = 70-0.57 = 69.43%
    Hence, the mean literacy rate is 69.43%.
    Question 10
    CBSEENMA10009107

    The following table shows the ages of the patients admitted in a hospital during a year.

    Find the mode and mean of the data given below. Compare and interpret the measures of central tendency.

    Age (in yrs)

    No. of patients

    5-15

    6

    15-25

    11

    25-35

    21

    35-45

    23

    45-55

    14

    55-65

    5

     

    Solution

    Case I: Finding the mode
    Here, the maximum class frequency is 23 and the class corresponding to frequency is 35-45.
    So, the modal class is 35-45.
    Thus, we have
            Modal class = 35 - 45 
                  = 35 
                  fi = 23, f0 = 21,  f2 = 14
    and         h = 10
    Now, substituting these values in the formula of mode, we get
                 Mode space equals space straight l space plus space open square brackets fraction numerator straight f subscript straight i minus straight f subscript 0 over denominator 2 straight f subscript 1 minus straight f subscript 0 minus straight f subscript 2 end fraction close square brackets cross times straight h equals 35 plus open square brackets fraction numerator 23 minus 21 over denominator 2 cross times 23 minus 21 minus 14 end fraction close square brackets cross times 10
space space space space space space space space space equals space 35 plus open square brackets fraction numerator 2 over denominator 46 minus 35 end fraction close square brackets cross times 10 equals 35 plus 2 over 11 straight x space 10 space equals 35 plus 20 over 11 equals 35 plus 1.818 equals 36.818.
    Case II : Finding the median

    Now,
    open parentheses top enclose straight x close parentheses space equals space A space plus space open parentheses fraction numerator begin display style sum from blank to blank of end style f subscript i u subscript i over denominator begin display style sum from blank to blank of end style f subscript i end fraction close parentheses cross times h equals 30 plus 43 over 80 x 10 equals 30 plus 43 over 8 equals 30 plus 5.373 equals 35.373

    Hence, the mode of the given data is 36.818 while mean is 35.373.

    Interpretation : Maximum number of patients admitted in the hospital are of the age 36.818 years, while an average age of patients admitted to the hospital is 35.373 yrs.


    Question 11
    CBSEENMA10009108

    The following data gives the information on the observed life-times (in hours) of 225 electrical components.

    Life time (in hours)

    Frequency

    0-20

    10

    20-40

    35

    40-60

    52

    60-80

    61

    80-100

    38

    100-120

    29

    Determine the modal lifetimes of the components.

    Solution
    Here, the maximum class frequency is 61 and the class corresponding to frequency is 60-80. So the modal class is 60-80.
    Thus, we have 
                              Modal class = 60-80
                                     i = 60, h = 20,  fi=61
                                    f0 = 52 and f= 38
    Now, substituting these in the formula of mode, we get
                            Mode space equals space straight i space plus space open square brackets fraction numerator straight f subscript straight i minus straight f subscript 0 over denominator 2 straight f subscript straight i minus straight f subscript 0 minus straight f subscript 2 end fraction close square brackets space straight x space straight h
space space space space space space space space equals space 60 space plus space open square brackets fraction numerator 61 minus 52 over denominator 2 space straight x space 61 space minus 52 minus 38 end fraction close square brackets space straight x space 20
space space space space space space space space equals space 60 plus open square brackets fraction numerator 9 over denominator 122 minus 90 end fraction close square brackets space straight x space 20
space space space space space space space space equals space 60 space plus space 9 over 32 space straight x space 20 space equals space 60 space plus 45 over 8
space space space space space space space space equals space 60 plus space 5.625 space equals space 65.625
    Hence, the modal life-times of the components are 65.625 hrs.
    Question 12
    CBSEENMA10009109

    The following data gives the distribution of total monthly household expenditure of 200 families of a village. Find the modal monthly expenditure of the families. Also, find the mean monthly expenditure:

    Expenditure (in Rs.)

    Number of families

    1000-1500

    24

    1500-2000

    40

    2000-2500

    33

    2500-3000

    28

    3000-3500

    30

    3500-4000

    22

    4000-4500

    16

    4500-5000

    7

     

    Solution

    Expenditure (in Rs.)

    Number of families

    1000-1500

    24

    1500-2000

    40

    2000-2500

    33

    2500-3000

    28

    3000-3500

    30

    3500-4000

    22

    4000-4500

    16

    4500-5000

    7

    Here, the maximum class frequency is 30 and the class corresponding to frequency is 3000-3500.
    So, the modal class = 1500-2000.
    Thus, we haveModal class = 1500-2000
    l = 1500
    f1 = 40, f0 = 24, f2 = 33 and h = 500
    Now, substituting these values in the formula of mode, we get
    Mode space equals space straight l plus open square brackets fraction numerator straight f subscript straight i minus straight f subscript 0 over denominator 2 straight f subscript straight i minus straight f subscript 0 minus straight f subscript 2 end fraction close square brackets straight x space straight h space equals space 1500 plus open square brackets fraction numerator 40 minus 24 over denominator 2 cross times 40 minus 24 minus 33 end fraction close square brackets space straight x space 500
space space space space space space space space equals space 1500 plus open parentheses fraction numerator 16 over denominator 80 minus 57 end fraction close parentheses cross times 500 equals 1500 plus fraction numerator 16 cross times 500 over denominator 23 end fraction equals 1500 plus 8000 over 23 equals 1500 plus 347.83 equals 1847.83

    Hence, the modal monthly expenditure of the families is Rs. 1847.83.
    Finding Mean:

    Here, we have  sum from blank to blank of f subscript i equals 200 comma space sum from blank to blank of f subscript i u subscript i equals space minus 235 comma space h space equals space 500 space a n d space A space equals space 3250
    Now,               top enclose straight x space equals space A space plus space open parentheses fraction numerator begin display style sum from blank to blank of end style f subscript i u subscript 1 over denominator begin display style sum from blank to blank of end style f subscript i end fraction close parentheses space x space h space equals space 3250 space plus space open parentheses fraction numerator negative 235 over denominator 200 end fraction close parentheses space x space 500 space equals space 3250 space minus fraction numerator 235 space x space 5 over denominator 2 end fraction equals 3250 minus 1175 over 2
           
                         = 3250 - 587.50 = 2662.50
    Hence, the mean monthly expenditure is Rs. 2662.50.

    Question 13
    CBSEENMA10009110

    The following distribution gives the state-wise teacher-student ratio in higher secondary schools of India. Find the mode and mean of this data. Interpret the two measures.

    Number of students per teacher

    Number of states/U.T.

    15-20

    3

    20-25

    8

    25-30

    9

    30-35

    10

    35-40

    3

    40-45

    0

    45-50

    0

    50-55

    2

    Solution

    Number of students per teacher

    Number of states/U.T.

    15-20

    3

    20-25

    8

    25-30

    9

    30-35

    10

    35-40

    3

    40-45

    0

    45-50

    0

    50-55

    2

    Here, the maximum class frequency is 10 and the class corresponding to frequency is 30-35.
    So, the modal class = 30-35
    Thus, we have
    modal class = 30-35, l = 30, f1 = 10, f0 = 9, f2 = 3 and h = 5
    Now, substituting these values in the formula of mode, we get
    Mode space equals space l plus open square brackets fraction numerator straight f subscript 1 minus straight f subscript 0 over denominator 2 straight f subscript 1 minus straight f subscript 0 minus straight f subscript 2 end fraction close square brackets space straight x space straight h space equals space 30 plus open square brackets fraction numerator 10 minus 9 over denominator 2 space straight x space 10 minus 9 minus 3 end fraction close square brackets space straight x space 5 space equals space 30 plus 1 over 8 straight x 5 equals 30 plus 5 over 8 equals 30 plus 0.6 equals 30.6
    Finding Mean:

    Here, we have   sum from blank to blank of f subscript 1 equals 35 comma space sum from blank to blank of f subscript 1 u subscript 1 equals negative 58 comma space h space equals space 3 space a n d space A space equals space 37.5
    Now,               top enclose straight x space equals space A plus fraction numerator begin display style sum from blank to blank of end style f subscript 1 u subscript 1 over denominator begin display style sum from blank to blank of end style f subscript 1 end fraction x space h equals 37.5 plus open parentheses fraction numerator negative 58 over denominator 35 end fraction close parentheses x space 5 space equals 37.5 space minus 8.3 equals 29.2
    Interpretation : Most stales/U.T have a student teacher ratio of 30.6 and on an average, this ratio is 29.2.

    Question 14
    CBSEENMA10009111

    The given distribution shows the number of runs scored by some top batsmen of the world in one-day international cricket matches.

    Runs scored

    Number of batsman

    3000-4000

    4

    4000-5000

    18

    5000-6000

    9

    6000-7000

    7

    7000-8000

    6

    8000-9000

    3

    9000-10000

    1

    10000-11000

    1

     

    Find the mode of the data.

    Solution

    Runs scored

    Number of batsman

    3000-4000

    4

    4000-5000

    18

    5000-6000

    9

    6000-7000

    7

    7000-8000

    6

    8000-9000

    3

    9000-10000

    1

    10000-11000

    1

     

    Here, maximum class frequency is 18 and the class corresponding to frequency is 4000-5000. So, the modal class = 4000-5000.
    Now,

     top enclose straight x space equals space A plus fraction numerator begin display style sum from blank to blank of end style f subscript i u subscript i over denominator begin display style sum from blank to blank of end style f subscript i end fraction x space h

    Thus, we have    Modal class = 4000-5000, l = 4000, f1 = 18, f0 = 4, f2 = 9 and h = 1000
    Now, substituting these values in the formula of mode, we get

    Mode space equals space straight l plus open square brackets fraction numerator straight f subscript 1 minus straight f subscript 0 over denominator 2 straight f subscript 1 minus straight f subscript 0 minus straight f subscript 2 end fraction close square brackets straight x space straight h
equals space 4000 plus open square brackets fraction numerator 18 minus 4 over denominator 2 space straight x space 18 minus 4 minus 9 end fraction close square brackets straight x 1000
equals 4000 plus 14000 over 23 equals 4000 plus 608.7 equals 4608.7
    Hence, the mode of the data is 4608.7.
    Question 15
    CBSEENMA10009112

    A student noted the number of cars passing through a spot on a road for 100 periods each of 3 minutes and summarised it in the table given below. Find the mode of the data :

    Numberof cars

    0-10

    10-20

    20-30

    30-40

    40-50

    50-60

    60-70

    70-80

    Frequency

    7

    14

    13

    12

    20

    11

    15

    8

     

    Solution

    Number of cars

    0-10

    10-20

    20-30

    30-40

    40-50

    50-60

    60-70

    70-80

    Frequency

    7

    14

    13

    12

    20

    11

    15

    8

    Here, maximum class frequency is 20 and the class corresponding to frequency is 40-50.
    So, the modal class = 40-50.
    Thus, we havemodal class = 40-50, l = 40, f1 = 20, f0 = 12, f2 = 11 and h = 10
    Now, substituting these values in the formula of mode, we get
    Mode space equals space l plus open square brackets fraction numerator straight f subscript 1 minus straight f subscript 0 over denominator 2 straight f subscript 1 minus straight f subscript 2 minus straight f subscript 2 end fraction close square brackets straight x space straight h space equals space 40 plus open square brackets fraction numerator 20 minus 12 over denominator 2 cross times 20 minus 12 minus 11 end fraction close square brackets space straight x space 10 space equals space 40 plus 80 over 17 equals 40.4.7 space equals space 44.7
    Hence, the mode of the data is 44.7 cars.

    Question 16
    CBSEENMA10009113

    The following frequency distribution gives the monthly consumption of electricity of 68 consumers of a locality. Find the median, mean and mode of the data and compare them.

    Monthly consumption of electricity (in units)

    Number of consumers

    68-85

    4

    85-105

    5

    105-125

    13

    125-145

    20

    145-165

    14

    165-185

    8

    185-205

    4

     

    Solution

    Finding Median:

    Monthly consumption of electricity (in units)

    Number of consumers

    Cumulative frequency

    68-85

    4

    4

    85-105

    5

    9

    105-125

    13

    22

    125-145

    20

    42

    145-165

    14

    56

    165-185

    8

    64

    185-205

    4

    68

     

    n = 68

     

    We have n = 68. So,  open parentheses straight n over 2 close parenthesesth observation = 34th observation.
    So, median lies in the group of 125-145 i.e. median class = 125-145
    Now, we have median class = 125.145, i = 125, straight n over 2 = 34, cf = 22, f = 20 and h  =20
    Substituting these values in the formula of median, we get

    Median space equals space l italic space plus space open square brackets fraction numerator begin display style straight n over 2 end style minus cf over denominator straight f end fraction close square brackets straight x space straight h space equals space 125 plus open square brackets fraction numerator 34 minus 22 over denominator 20 end fraction close square brackets space straight x space 20 space equals space 125 space plus space 12 space equals space 137 space units.Finding Mean:


    Here. we have                        sum from blank to blank of f subscript i space equals space 68 comma space sum from blank to blank of f subscript i u subscript i space end subscript equals 7 comma space h space equals space 20 space a n d space A space equals space 135

    Now,                                       top enclose straight x space equals space A plus fraction numerator begin display style sum from blank to blank of end style f subscript i u subscript i over denominator begin display style sum from blank to blank of end style f subscript i end fraction space x space h
                                                  
                                                   equals space 135 plus 7 over 68 straight x space 20
equals space 135 plus 35 over 17 equals 135 plus 2.05 equals 137.05

    Finding Mode :
    Here, the maximum class frequency is 20 and the class corresponding to frequency is 125-145.
    So, the modal class = 125-145.
    Thus, we have, Modal class = 125-145, l = 125, f1 = 20, f0 = 13, f2= 14, and h = 20
    Now, substituting these values in the formula of mode, we get
    Mode space equals space l plus open square brackets fraction numerator straight f subscript straight i minus straight f subscript 0 over denominator 2 straight f subscript 1 minus straight f subscript 0 minus straight f subscript 2 end fraction close square brackets space straight x space straight h space equals space 125 plus open square brackets fraction numerator 20 minus 13 over denominator 2 space straight x space 20 minus 13 minus 14 end fraction close square brackets space straight x space 20 space equals space 125 plus open square brackets fraction numerator 7 over denominator 40 minus 17 end fraction close square brackets space straight x space 20
space space space space space space space space equals space 125 plus 140 over 13 equals 125 plus 10.76 equals 135.76 space units.
    Comparison: On comparison, we find that the three measures are approximately the same in this case.

    Question 17
    CBSEENMA10009114

    If the median of the distribution given below is 28.5, find the values of x and y.

    Class interval

    Frequency

    0-10

    5

    10-20

    x

    20-30

    20

    30-40

    15

    40-50

    y

    50-60

    5

    Total

    x = 60

     

    Solution

    Forming the cumulative frequency table, we have

    Class Interval (C.I.)

    Frequency

    c.f.

    0-10

    5

    5

    10-20

    x

    5 + x

    20-30

    20

    25+ x

    30-40

    15

    40 + x

    40-50

    y

    40 + x + y

    50-60

    5

    45 + x + y

    Total

    60

     

    It is given that                                  n = 60
    rightwards double arrow space                                    45 + x + y = 60
    rightwards double arrow                                             x + y = 15
    It is given that median is 28.5
    So, the median lies in the group of 30-40
    i.w.                          median class = 20 - 30
    Thus, we have Median Class = 20 - 30
                                      l italic space equals space 20 comma space space straight n over 2 equals 30 comma space straight c. straight f. space equals space 5 plus straight x comma space straight h space equals 10 space and space straight f space equals 20
    Now, substituting these values in the formula of Median, we get
      space space space space space space space space space space space space space space space space space space space space space space space space space space space space space space space space space space space space space space Median space equals space straight l plus open square brackets fraction numerator begin display style straight n over 2 minus cf end style over denominator straight f end fraction close square brackets straight x space straight h
rightwards double arrow space space space space space space space space space space space space space space space space space space space space space space space space space space space space space space space space space space space 28.5 space equals space 20 plus open square brackets fraction numerator 30 minus left parenthesis 5 plus straight x right parenthesis over denominator 20 end fraction close square brackets space straight x space 10
rightwards double arrow space space space space space space space space space space space space space space space space space space space space space space space space space space space space space space space space space space space 28.5 space equals space 20 plus open square brackets fraction numerator 30 minus 5 minus straight x over denominator 20 end fraction close square brackets straight x space 10
rightwards double arrow space space space space space space space space space space space space space space space space space space space space space space space space space space space space space space space space space space space 28.5 space equals space 20 plus open parentheses fraction numerator 25 minus straight x over denominator 20 end fraction straight x 10 close parentheses
rightwards double arrow space space space space space space space space space space space space space space space space space space space space space space space space space space space space space space space space space space space 28.5 space equals space 20 space plus space fraction numerator 25 minus straight x over denominator 2 end fraction
rightwards double arrow space space space space space space space space space space space space space space space space space space space space space space space space space space space space space space space space space space space 28.5 space equals space fraction numerator 40 plus 25 minus straight x over denominator 2 end fraction
      rightwards double arrow                              57 = 65 - x
      rightwards double arrow                              x = 65 - 57
      rightwards double arrow                              x = 8
    Putting the value of x = 8 in (i), we get
                               x + y = 15
    rightwards double arrow                       8 + y = 15
    rightwards double arrow                           y = 15 - 8 = 7
    Hence,         x = 8 and y = 7 
      
    Question 18
    CBSEENMA10009115

    A life insurance agent found the folio wing data for distribution of ages of 100 policy holders. Calculate the median age, if policies are only given to persons having age 18 yrs. onwards but less than 60 years.

    Age (in yrs.)

    No. of policy holders

    below 20

    2

    below 25

    6

    below 30

    24

    below 35

    45

    below 40

    78

    below 45

    89

    below 50

    92

    below 55

    98

    below 60

    100

    Solution

    Age (in yrs)

    No. of policy holders

    c.f.

    below 20

    2

    2

    20-25

    4

    6

    25-30

    18

    24

    30-35

    21

    45

    35-40

    33

    78

    40-45

    11

    89

    45-50

    3

    92

    50-55

    6

    98

    55-60

    2

    100

     

    n = 100

     

    Here, middle term is open parentheses 100 over 2 close parentheses th or 50 th observation, which lies in class 35 - 45.
    Thus, we have Median class = 35 - 40 = 35,  straight n over 2 space equals space 50 space c. f. space equals space 45 comma space f space equals space 33 space a n d space h space equals space 5
    Putting these values in the formula of median, we get
    Median space equals space straight l plus open square brackets fraction numerator begin display style straight n over 2 end style minus cf over denominator straight f end fraction close square brackets space straight x space straight h space equals 35 space plus space open square brackets fraction numerator 50 minus 45 over denominator 33 end fraction close square brackets space straight x space 5 space equals space 35 space plus space fraction numerator 5 space straight x space 5 over denominator 33 end fraction equals 35 space plus 25 over 33 equals 35 plus 0.75 equals 35.75
    Hence, the median age is 35.75 yrs.    
    Question 19
    CBSEENMA10009116

    The lengths of 40 leaves of a plant are measured correct to the nearest millimetre and the data obtained is represented in the following table:

    Length (in nun)

    Number of leaves

    118-126

    3

    127-135

    5

    136-144

    9

    145-153

    12

    154-162

    5

    163-171

    4

    172-180

    2

    Find the median length of the leaves.

    Solution

    We shall first convert the given data to continuous classes. Then, the data become

    Length (in mm)

    Number of leaves

    Cumulative frequency

    117.5-126.5

    3

    3

    126.5-135.5

    5

    8

    135.5-144.5

    9

    17

    144.5-153.5

    12

    29

    153.5-162.5

    5

    34

    162.5-171.5

    4

    38

    171.5-180.5

    2

    40

       

    n = 40

     

    We have n = 40. So,  open parentheses straight n over 2 close parentheses t h  observation = 20th observation.
    So, median lies in the group of 144.5 - 153.5 i.e.    median class = 144.5-153.5
    Thus, we have
    Median class = 144.5 - 153.5 = 144.5, straight n over 2=20, cf = 17, f=12 and h =10
    Substituting these values in the formula of median we get
    Median space equals straight l plus open square brackets fraction numerator begin display style straight n over 2 end style minus cf over denominator straight f end fraction close square brackets straight x space straight h space equals space 144.5 space plus open square brackets fraction numerator 20 minus 17 over denominator 12 end fraction close square brackets space straight x space 9
space space space space space space space space space space space equals space 144.5 space plus 3 over 12 straight x space 9 space equals space 144.5 space plus 9 over 4
space space space space space space space space space space space equals space 144.5 space plus space 2.25 space equals space 146.75 space mm
    Question 20
    CBSEENMA10009117

    The following table gives the distribution of the life-time of 400 new lamps.

    Life-time (hrs.)

    No. of lamps

    1500-2000

    14

    2000-2500

    56

    2500-3000

    60

    3000-3500

    86

    3500-4000

    74

    4000-4500

    62

    4500-5000

    48

    Solution

    Forming the cumulative frequency table, we get

    Life-time (hrs.)

    No. of lamps (f)

    c.f

    1500-2000

    14

    14

    2000-2500

    56

    70

    2500-3000

    60

    130

    3000-3500

    86

    216

    3500-4000

    74

    290

    4000-4500

    62

    352

    4500-5000

    48

    400

     

    n = 400

     

    We have n = 400, so,  straight n over 2 th observation = 200th observation.
    So, median lies in the group of 3000-3500. i.e.,Median class = 3000 - 3500
    Now, we have                   Median class = 3000-3500
                                                            l =  300, straight n over 2 = 200,  cf = 130, f = 86 and h = 500 
    Substituting these values in the formula of Median, we get
    Median space space equals space straight l plus open square brackets fraction numerator begin display style straight n over 2 end style minus cf over denominator straight f end fraction close square brackets space straight x space straight h
space space space space space space space space space space space space equals space 3000 plus open square brackets fraction numerator 200 minus 130 over denominator 86 end fraction close square brackets space straight x space 500
space space space space space space space space space space space space equals space 3000 plus 70 over 86 straight x space 500 space equals space 3000 plus 35000 over 86
              = 3000 + 406.97 = 3406.97 

    Hence, the median life time of a lamps = 3406.97.
    Question 21
    CBSEENMA10009118

    100 surnames were randomly picked up from a local telephone directory and the frequency distribution of the number of letters in the English alphabets in the surnames was obtained as follows:

    Number of letters

    1-4

    4-7

    7-10

    10-13

    13-16

    16-19

    Number of surnames

    6

    30

    40

    16

    4

    4

    Determine the median number of letters in the surnames. Find the mean number of letters in the surnames? Also, find the modal size of the surnames.

    Solution

    Number of letters

    Number of surnames

    Cumulative frequency

    1-4

    6

    6

    4-7

    30

    36

    7-10

    40

    76

    10-13

    16

    92

    13-16

    4

    96

    16-19

    4

    100

     

    x = 100

     

    We have n = 100, so  open parentheses straight n over 2 close parentheses th observation = 50th observation.

    So, median lies in the group of 7-10.
    i.e.                                     Median class = 7 - 10
    Now, we have                      Median class = 70 - 10 l = 7, straight n over 2 equals 50, cf = 36, f = 40 and h =3
    Now, substituting these values in the formula of median, we get
    Median space equals space space straight l plus open square brackets fraction numerator begin display style straight n over 2 end style minus cf over denominator straight f end fraction close square brackets space straight x space straight h space equals space 7 plus open square brackets fraction numerator 50 minus 36 over denominator 40 end fraction close square brackets space straight x 3 space equals space 7 plus 21 over 20 equals 7 plus 1.05 space equals space 8.05
    Hence, the median number of letters in the surnames is 8.05.

    Question 22
    CBSEENMA10009119

    The distribution below gives the weights of 30 students of a class. Find the Median weight of the students.

    Weight (in k.g)

    40-45

    45-50

    50-55

    55-60

    60-65

    65-70

    70-75

    No. of students (f)

    2

    3

    8

    6

    6

    3

    2

    Solution

     Forming the cumulative frequency table we get,

    Weight (in k.g)

    No. of students (f)

    c.f

    40-45

    2

    2

    45-50

    3

    5

    50-55

    8

    13

    55-60

    6

    19

    60-65

    6

    25

    65-70

    3

    28

    70-75

    2

    30

     

    n = 30

     

    We have n = 30, So,open parentheses straight n over 2 close parentheses  th observation = 15th observation.
    so, median lies in the group of 55 - 60
    i.e. median class                 = 55 - 60
    Now, we have Median class  = 55 - 60
                                           = 55, straight n over 2=15, cf = 13, f = 6 and h = 5
    Substituting these values in the formula of Median, we get
    Median space equals space l italic space plus open square brackets fraction numerator begin display style straight n over 2 end style minus cf over denominator straight f end fraction close square brackets space straight x space straight h space equals space 55 plus open square brackets fraction numerator 15 minus 13 over denominator 6 end fraction close square brackets space straight x space 5 space equals space 55 plus 2 over 6 space straight x space 5 space equals space 55 plus 5 over 3 equals 55 plus 1.67 equals 56.67
    Hence, the median weight of the students = 56.67 kg.
    Question 23
    CBSEENMA10009120

    The following distribution gives the daily income of 50 workers of a factory.

    Daily income (in Rs.)

    100-120

    120-140

    140-160

    160-180

    180-200

    Number of workers

    12

    14

    8

    6

    10

    Convert the distribution above to a less than type cumulative frequency distribution, and draw its ogive.

    Solution

     Cf distribution table

    Daily income (in Rs.)

    Number of workers

    Less than 120

    12

    Less than 140

    26

    Less than 160

    34

    Less than 180

    40

    Less than 200

    50

    Now, we plot the points : (120,12), (140, 26), (160,34), (180,40), (200,50).

    Question 24
    CBSEENMA10009121

    Durine the medical check-up of 35 students of a class, their weight were recorded as follows:

    Weight (in kg)

    Number of students

    Less than 38

    0

    Less than 40

    3

    Less than 42

    5

    Less than 44

    9

    Less than 46

    14

    Less than 48

    28

    Less than 50

    32

    Less than 52

    35

    During a less than type ogive for the given data. Hence obtain the median weight from the graph - and verify the result by using the formula.

    Solution

    Here,   straight n over 2 space equals space 35 over 2 space equals space 17.5

    Locatae 17.5 on the y-axis. From this point, draw a line parallel to the x-axis cutting the curve at a point. From this point, draw a perpendicular to the x-axis. The point of intersection of this perpendicular with the x-axis determines the median of the given data as 46.5 kg.

    Median weight by using the formula :

    Weight (in k.g)

    No. of students

    Cumulative frequency

    0-38

    0

    0

    38-40

    3

    3

    40-42

    2

    5

    42-44

    4

    9

    44-46

    5

    14

    46-48

    14

    28

    48-50

    4

    32

    50-52

    3

    35

     

    n = 35

     


    We have n = 35. So open parentheses straight n over 2 close parentheses th observation = 17.5th observation.

    So, median lies in the group of 46-48 i.e.Median class = 46-48

    Now, we have                       Median class  = 46 -48
                                                          = 46, straight n over 2 equals 17.5, cf = 14, f = 14 and h = 2
    Substituting these values in the formula of median, we get
    Median space equals space l plus open square brackets fraction numerator begin display style straight n over 2 end style minus cf over denominator straight f end fraction close square brackets space straight x space straight h space equals space 46 plus open square brackets fraction numerator 17.5 minus 14 over denominator 14 end fraction close square brackets cross times 2 equals 46 plus 1 half equals 46.5 space kg
    Verification : We find that the median weight obtained from the graph is the same on the median weight obtained by using the formula.
    Question 26
    CBSEENMA10009123

    A student draws a cumulative frequency curve for the marks obtained by 40 students of a class as shown below. Find the median marks obtained by the student of the class.


    Solution

    Here   n = 40 rightwards double arrow space straight n over 2 equals 40 over 2 equals 20

    Locate 20 on the y-axis, From this point, draw a line parallel to the x-axis. The point of intersection of this perpendicular with the x-axis gives the median of the data.

    Hence, the median marks obtained by the student of the class = 54 marks.


    Question 27
    CBSEENMA10009124

    Which measure of central tendency is given by the x-coordinate of the point of intersection of the 'more than ogive' and 'less than ogive'?

    Solution
    The median of a grouped data of central tendency is given by the x-coordinate of the point of intersection of the 'more than ogive' and 'less than ogive'.
    Question 28
    CBSEENMA10009125

    Write the formula for finding the median for a gouped or continuous frequency distribution.

    Solution
    Median space equals space straight l space plus open square brackets fraction numerator begin display style straight n over 2 end style minus cf over denominator straight f end fraction close square brackets space straight x space straight h, where the symbols have their usual meanings.
    Question 29
    CBSEENMA10009126

    What is the value of the median of the data using the graph in figure, of less than ogive and more than ogive?


    Solution
    From the figure, we can see that the two ogives are intersecting each other at a point. From this point, if we draw a perpendicular on the x-axis, the point at which it cuts the x-axis gives us the median. Hence, the median is 4.
    Question 30
    CBSEENMA10009127

    What is the value of the median of the data using the graph in figure, of less than ogive?






    Solution
    The graph shows that the total number of students is 100. Here n = 100 rightwards double arrowstraight n over 2 equals 50
    From the point corresponding to 50 on y-axis draw a line parallel to x-axis cutting the curve at P. From this point P draw a line parallel to y-axis meeting x-axis at the point M. The x-coordinate of M is 32.5 Hence, median is 32.5
    Question 31
    CBSEENMA10009128

    The wickets taken by a bowler in 10 cricket matches are as follows:
    2, 6, 4, 5, 0, 2, 1, 3, 2, 3
    Find the mode of the data.

    Solution
    Clearly, 2 is the number of wickets taken by the bowler in the maximum number (i.e.,3) or matches. So, the mode of the distribution is 2.
    Question 32
    CBSEENMA10009129

    Write the formula for finding the mode for a grouped or continous frequency distribution.

    Solution

    The mode for a grouped or continuous frequency distribution is given by

    Mode space equals space 1 plus open square brackets fraction numerator straight f subscript 1 minus straight f subscript 0 over denominator 2 straight f subscript 1 minus straight f subscript 0 minus straight f subscript 2 end fraction close square brackets cross times straight h comma
    where the symbols have their usual meanings.

    Question 33
    CBSEENMA10009130

    The wickets taken by a bowler in 10 cricket matches are as follows:
    3 6 4 5 0 2 1 3 2 3
    Find the mode of the data

    Solution
    Clearly 3 is the number of wickets taken by a bowler in the maximum numbers (i.e.) 3. So the mode of distribution is 3.
    Question 35
    CBSEENMA10009132

    Calculate the mode of the following data.
    4, 6, 7, 9, 12, 11, 13, 9, 13, 9, 9, 7, 8.

    Solution

    Let us form the frequency distribution table of the given data by arranging it in ascending or in descending order.

    x

    4

    6

    7

    8

    9

    11

    12

    13

    f

    1

    1

    2

    1

    4

    1

    1

    2

    We observe that the value of 9 occurs most frequently i.e., 4 times in the given set of observations.
    So, the mode is 9.

    Question 36
    CBSEENMA10009133

    The wicket taken by a bowler in 10 cricket matches arc as follows :
    2, 6, 4, 5, 0, 2, 11, 3, 2, 3.
    Find the mode of the data.

    Solution

    Let us form frequency distribution table.

    x

    0

    1

    2

    3

    4

    5

    6

    f

    1

    1

    3

    2

    1

    1

    1

    We observe that the value of 2 occurs most frequently i.e., 3 times in the given data.
    So, the mode is 2.
    Thus, the mode is defined as the most frequently occurring values.

    Question 37
    CBSEENMA10009134

    Calculate median of the following distribution.

    Marks (x)

    No. of students

    10

    2

    20

    8

    30

    16

    40

    26

    50

    20

    60

    16

    70

    7

    80

    4

    Solution

    Forming the cumulative frequency table, we have

    Marks (x)

    No. of students

    Cum. frequencies (c.f.)

    10

    2

    2

    20

    8

    10

    30

    16

    26

    40

    26

    52

    50

    20

    72

    60

    16

    88

    70

    7

    95

    80

    4

    99


    Here.     N = 99
    rightwards double arrow space space space space space space straight N over 2 equals 49.5
    We find that cumulative frequency just greater than straight N over 2 i.e., 49.5 is 52 and the value of x (variable) corresponding to 52 is 40 therefore, median = 40 Hence, median = 40.
    Question 38
    CBSEENMA10009135

    Calculate the median for the following:

    Heights (am)

    120

    121

    122

    123

    124

    125

    No. of students

    8

    12

    17

    14

    13

    6

    Solution

    Forming the cumulative frequency table, we have

    Height (x)

    No. of students (f)

    c.f.

    120

    8

    8

    121

    12

    20

    122

    17

    37

    123

    14

    51

    124

    13

    64

    125

    6

    70


    N = 70, which is even
    therefore   Here, 
                       N = 70
    rightwards double arrow space space space space space space space space space space space space space space space space space straight N over 2 equals 35
    We find that cumulative frequency just greater than straight N over 2 i.e., 35 is 37 and the value of x (variable) corresponding to 37 is 122.
    Therefore, median = 122.
    Question 39
    CBSEENMA10009136

     Write the median class of the following distribution.    

    Classes

    Frequency

    0—10

    4

    10—20

    4

    20—30

    8

    30-40

    10

    40—50

    12

    50—60

    8

    60—70

    4

    Solution

    Classes

    f

    c.f

    0—10

    4

    4

    10—20

    4

    8

    20—30

    8

    16

    30—40

    10

    26

    40—50

    12

    38

    50—60

    8

    46

    60—70

    4

    50


    Here,  n = 50,      therefore    straight n over 2 equals 25
    Now, 30—40 is the classes whose c.f 26 is greater than (and nearest to)  straight n over 2 space i. e. comma space 25
    Hence, 30—40 is the median class.
    Question 42
    CBSEENMA10009139
    Question 46
    CBSEENMA10009143
    Question 48
    CBSEENMA10009145

    The marks obtained by 40 students of class X of a certain school in Science paper consisting of 10 marks are presented in the table below. Find the mean marks obtained by the students.

    Marks obtained(xi)

    5

    6

    7

    8

    9

    Number of students (fi)

    4

    8

    14

    11

    3

    Solution

    Marks Obtained (xi)

    Number of students (fi)

    fixi

    5

    4

    20

    6

    8

    48

    7

    14

    98

    8

    11

    88

    9

    3

    27

     

    Σfi = 40

    Σfixi = 281


    Now,
    top enclose straight x space equals fraction numerator begin display style stack sum f subscript i x subscript i with blank below and blank on top end style over denominator begin display style sum from blank to blank of end style f subscript i end fraction equals 281 over 40 equals 7.025
    Hence, the mean marks obtained is 7.025.
    Question 49
    CBSEENMA10009146

    Consider the following distribution of daily wages of 50 workers of a factory.

    Daily wages

    100-120

    120-140

    140-160

    160-180

    180-200

    No. of workers

    12

    14

    8

    6

    10

    Find the mean daily wages of the workers.

    Solution

    Daily wages (C.I.)

    No. of workers (fi)

    Class-Mark (xi)

    fi xi

    100 - 120

    12

    110

    1320

    120 - 140

    14

    130

    1820

    140 - 160

    8

    150

    1200

    160 - 180

    6

    170

    1020

    180 - 200

    10

    190

    1900

     

    Σfi = 50

     

    Σfi xi = 7260


    Now,
    top enclose straight x space equals space fraction numerator begin display style sum from blank to blank of end style f subscript i x subscript i over denominator begin display style sum from blank to blank of end style f subscript i end fraction equals 7260 over 50 equals 145.2
    Hence, the mean daily wages of the workers be 145.2.
    Question 50
    CBSEENMA10009147

    Find the mean of the following frequency distribution.

    Class-Interval

    0-8

    8-16

    16-24

    24-32

    32-40

    Frequency

    8

    10

    15

    9

    8

     

    Solution

    Class-Interval (C.I.)

    Frequency (fi)

    Class-mark (xi)

    (fixi)

    0 - 8

    8

    4

    32

    8 - 16

    10

    12

    120

    16 - 24

    15

    20

    300

    24 - 32

    9

    28

    252

    32 - 40

    8

    36

    288

     

    Σfi = 50

     

    Σfixi = 992

    Now,
    top enclose X space equals space fraction numerator begin display style sum from blank to blank of end style f subscript i x subscript i over denominator begin display style sum from blank to blank of end style f subscript i end fraction equals 992 over 50 equals 19.84

    Hence, (he mean of frequency distriution is 19.84.

    If the values of x or f are large, the calculation of mean by the direct method is quite lengthy and time consuming so, to minimize the time involved in calculation, we prefer assumed mean method.

    Question 51
    CBSEENMA10009148

    Find the value of P, if the mean of the following distribution is 18.

    x:

    13

    15

    17

    19

    20 + p

    23

    f:

    8

    2

    3

    4

    5p

    Solution

    xi

    fi

    fixi

    13

    8

    104

    15

    2

    30

    17

    3

    51

    19

    4

    76

    20 + p

    5p

    5p (20 + p)

    23

    6

    138

     

    Σfi = 23 + 5p

    Σfixi = 399 + 5p2 + 100p

     

    Here, we have                    top enclose straight x equals 18 comma space space sum from blank to blank of straight f space equals space 23 space plus space 5 straight p space and space sum from blank to blank of straight f subscript straight i straight x subscript straight i space equals 399 plus 5 straight p squared plus 100 straight p
    Now,                                 top enclose straight x space equals space fraction numerator begin display style sum from blank to blank of end style f subscript i x subscript i over denominator begin display style sum from blank to blank of end style f subscript i end fraction equals fraction numerator 5 p squared plus 100 p plus 399 over denominator 5 p plus 23 end fraction
    rightwards double arrow                                     18 equals fraction numerator 5 straight p squared plus 10 straight p plus 399 over denominator 5 straight p plus 23 end fraction
    rightwards double arrow                      18(5p + 23) = 5p2 + 100p + 399
    rightwards double arrow                      90p + 414 = 5p2 + 100p + 399
    rightwards double arrow                      5p2 + 10p - 15 = 0
    rightwards double arrow                      p2 + 2p - 3 = 0
    rightwards double arrow                      p2 + 3p - p - 3 = 0
    rightwards double arrow                     p(p + 3) - 1 (p + 3) = 0
    rightwards double arrow                     (p - 1) (p + 3) = 0
    rightwards double arrow                      p - 1 = 0 or p + 3 = 0
    Since                   p =  1 or  p = - 3 neglected
    Therefore,            p = 1 

    Question 52
    CBSEENMA10009149

    The following table shows markes scored by 140 students in a examinations.

    Marks

    0-10

    10-20

    20-30

    30-40

    40-50

    No. of Sutdents

    20

    24

    40

    36

    20

     

    Calculate mean marks by using assumed mean method.

    Solution

    Marks (C.I.)

    No. of students (fi)

    Class mark (xi)

    di = xi - 25

    fidi

    0-10

    20

    5

    - 20

    -400

    10-20

    24

    15

    -10

    -240

    20-30

    40

    25=A

    0

    0

    30-40

    36

    35

    10

    360

    40-50

    20

    45

    20

    400

     

    Σfi = 140

       

    Σfidi = 120

    Now,
    top enclose straight x space equals space A plus fraction numerator begin display style stack sum f subscript i d subscript i with blank below and blank on top end style over denominator begin display style sum from blank to blank of end style f subscript i end fraction
equals space 25 space plus space fraction numerator 120 space over denominator 140 end fraction
equals space 25 plus 0.86 equals space 25.86
    Hence, the mean marks obtained by 140 students is 25.86.

    Question 53
    CBSEENMA10009150

    Find the mean of the following data

    Class-lnterval

    15-25

    25-35

    35-45

    45-55

    55-65

    Frequency

    60

    35

    22

    18

    15

    Solution

    C.I.

    fi

    Mid-point (xi)

    di = xi - 40

    fidi

    15-25

    60

    20

    -20

    -1200

    25-35

    35

    30

    -10

    -350

    35-45

    22

    40 = A

    0

    0

    45-55

    18

    50

    10

    180

    55-65

    15

    60

    20

    300

       

    Σfi = 150

     

    Σfidi = -1070


    Now,
    top enclose x space equals space A plus fraction numerator begin display style sum from blank to blank of end style f subscript i d subscript i over denominator begin display style sum from blank to blank of end style f subscript i end fraction
equals space 40 space plus open parentheses fraction numerator negative 1070 over denominator 150 end fraction close parentheses equals 40 minus 7.13 equals 32.87
    Hence, the mean of the data is 32.87.
    Question 54
    CBSEENMA10009151

    The following table represents the wages of 50 workers in a factory.

    Wages (in Rs.)

    1450

    1475

    1500

    1525

    1550

    No. of workers

    12

    13

    7

    10

    8


    Find the average wage of a workers.

    Solution

    Wages (Rs.) (xi)

    No. of workers (fi)

    di = xi-500

    fidi

    1450

    12

    -50

    -600

    1475

    13

    -25

    -325

    1500 = A

    7

    0

    0

    1525

    10

    25

    250

    1550

    8

    50

    400

     

    Σfi = 50

     

    Σfidi=-275


    Now,
    top enclose straight x space equals space A plus fraction numerator begin display style sum from blank to blank of end style f subscript i d subscript i over denominator begin display style sum from blank to blank of end style f subscript i end fraction
equals space 1500 plus open parentheses fraction numerator negative 275 over denominator 50 end fraction close parentheses equals 1500 minus 5.5 space equals 1494.50
    Hence, the average wage of a worker is 1494.50.
    Question 55
    CBSEENMA10009152

    The following table represents the expenditure on the usage of water by 70 families in a locality. Find the average expenditure per family.

    Exp. water (in Rs.)

    15-20

    20-25

    25-30

    30-35

    35-40

    40-45

    45-50

    50-55

    No. of families

    7

    8

    7

    8

    10

    15

    7

    8

     

    Solution

    Here,  we have
                                 A (assumed mean) = 37.5,   sum from blank to blank of f subscript i equals 70 comma space space space sum from blank to blank of f subscript i u subscript i equals negative 21 space a n d space h space equals space 5

    Now,                     top enclose straight x equals A plus fraction numerator begin display style sum from blank to blank of end style f subscript i u subscript i over denominator begin display style sum from blank to blank of end style f subscript i end fraction cross times h equals 37.5 plus open parentheses fraction numerator negative 21 over denominator 70 end fraction close parentheses cross times 5 equals 37.5 minus 21 over 14 equals 27.5 minus 1.5 equals 3.6
    Hence, the average expenditure per family is 36.
    Question 57
    CBSEENMA10009154

    The mean of the following frequency distribution is 62.8 and the sum of all frequencies is 50. Compute the missing frequency f1 and f2.

    C.I.

    0-20

    20-40

    40-60

    60-80

    80-100

    100-120

    Total

    f

    5

    f1

    10

    f2

    7

    8

    50

    Solution

    Here, we have               sum from blank to blank of straight f subscript straight i straight u subscript straight i space equals space 28 space plus space straight f subscript 2 minus straight f subscript 1
                                     
                                       top enclose straight x space equals space 62.8 space a n d space h space equals space 20 space a n d space sum from blank to blank of f subscript i equals 50
    It is given that,             sum from blank to blank of f subscript i equals 50
    rightwards double arrow                              30 + f1 + f  = 50
    rightwards double arrow                              f+ f   = 20
    Now,                           top enclose straight x space equals space A plus fraction numerator begin display style sum from blank to blank of f subscript i u subscript i end style over denominator begin display style sum from blank to blank of f subscript i end style end fraction cross times h
    rightwards double arrow                           62.8 space equals space 50 space plus space fraction numerator 28 minus straight f subscript 1 plus straight f subscript 2 over denominator 50 end fraction cross times 20
    rightwards double arrow                           62.8 space equals fraction numerator 1 left parenthesis 28 minus f subscript 1 plus f subscript 2 right parenthesis over denominator 5 end fraction plus 50 
    rightwards double arrow                           62.8 space equals fraction numerator 56 minus 2 straight f subscript straight i plus 2 straight f subscript 2 plus 250 over denominator 5 end fraction
    rightwards double arrow                            314 = -2f+ 2f2 + 306 rightwards double arrow  2f1- 2f2 = -8
    Adding (i) and (ii), we get            f - f2  = -4
    Putting this value in (i), we get    2f1 = 16 rightwards double arrow f = 8
    Hence,                                       f1 = 8,   f2 = 12

     
    Question 58
    CBSEENMA10009155

    The mean of the following frequency distribution is 132 and the sum of observations is 50. Find the missing frequency f1 and f2.

    Class

    0-40

    40-80

    80-120

    120-160

    160-200

    200-240

    Freq.

    4

    7

    f1

    12

    f2

    9

    Solution

    Here, we have                     sum from blank to blank of f subscript i space equals space 50 comma space sum from blank to blank of f subscript i u subscript i space equals space 2 f subscript 2 space plus space 24 comma space A (assumed mean) = 100
                                              
                                              top enclose straight x space equals space 132 space a n d space h space equals space 40

    It is given that                      sum from blank to blank of f subscript i space equals space 50
    rightwards double arrow                           f1 + f2 + 32 = 50
    rightwards double arrow                              f+ f = 18
    Now,                       top enclose straight x space equals space A plus fraction numerator begin display style sum from blank to blank of end style f subscript i u subscript i over denominator begin display style sum from blank to blank of end style f subscript i end fraction cross times h
    rightwards double arrow                       132 = 100 + fraction numerator 24 plus 2 straight f subscript 2 over denominator 50 end fraction cross times 40
    rightwards double arrow                        132 = 100 + fraction numerator 4 left parenthesis 2 straight f subscript 2 plus 24 right parenthesis over denominator 5 end fraction
    rightwards double arrow                       132 = fraction numerator 500 plus 8 straight f subscript 2 plus 96 over denominator 5 end fraction
    rightwards double arrow                    132 x 5 = 8f+ 596
    rightwards double arrow                     660 = 8f+ 596
    rightwards double arrow                      8f = 64
    rightwards double arrow                       f= 8
    Putting this value in (i), we get
                         straight f subscript 1 plus 8 space equals space 18
    rightwards double arrow                f = 10
    Hence.           f= 10,  f2 = 8

    Question 59
    CBSEENMA10009156

    The mean of the following frequency table is 53. But the frequencies f1 and f2 in the classes 20-40 and 60-80 arc missing. Find the missing frequencies.   

    Age (in years)

    0-20

    20-40

    40-60

    60-80

    80-100

    Total

    Number of people

    15

    f1

    21

    f2

    17

    100

    Solution

    Age (in yrs) class interval

    Mid values xi

    Frequency fi

    fi.xi

    0-20

    10

    15

    150

    20-40

    30

    f1

    30f1

    40-60

    50

    21

    1050

    60-80

    70

    f2

    70f2

    80-100

    90

    17

    1530

       

    Σfi = f1 + f2 + 53

    Σfixi = 2730 + 30f1 + 70f2


    It is given that :
                                                      sum from blank to blank of straight f subscript straight i space equals space 100
    rightwards double arrow                        15 + f+ 21 + f+ 17 = 100
    rightwards double arrow                                      f1 + f+ 53 = 100
    rightwards double arrow                                             f1 + f2 = 47
    Now,                                 top enclose straight x space equals space fraction numerator begin display style sum from blank to blank of end style f subscript i x subscript i over denominator begin display style sum from blank to blank of end style f subscript i end fraction
    rightwards double arrow                             53 = fraction numerator 2730 plus 30 straight f subscript 1 plus 70 straight f subscript 2 over denominator 100 end fraction
    rightwards double arrow                             53 x 100 = 2730 + 30f1 + 70f
    rightwards double arrow                             30f1 + 70f2  = 5300 - 2730
    rightwards double arrow                             30f1 + 70f2 = 2570
    rightwards double arrow                             3f1 + 7f= 257
    Thus, we have             f1 + f= 47
                                     3f1 + 7f= 257
    Multiplying equation (i) by 3, we get 
                                         3f1 + 7f2 = 141 
     and                               3f1 + 7f2 = 257 
    Subtracting (iv) from (iii), we get
                                  
                                     (3f1 + 3f2) - (3f1 + 7f2) = 141 - 257
    rightwards double arrow                             3f1 + 3f2 - 3f1-7f2 = -116
    rightwards double arrow                             -4f2 = -116
    rightwards double arrow                                f= 29
    Putting the value of fin (i), we get,
                              f+ f= 47 
    rightwards double arrow                     f+ 29 = 47
    rightwards double arrow                    f1= 1= 47 - 29
    rightwards double arrow                    f= 18
    Henc, f1 = 18 and f2= 29                     
     
                    
    Question 60
    CBSEENMA10009157

    Find the mode of the distribution from the following data.

    Class

    Frequcncy

    10-15

    3

    15-20

    2

    20-25

    10

    25-30

    7

    30-35

    20

    35-40

    5

    40-45

    8

    Solution

    Here, the maximum class frequency is 20 and the class corresponding to this frequency is 30-35, so the modal class is 30-35.
    Thus, we have
    Modal class = 30-35
    l (lower limit of modal class) 30
    Class size (h) = 5
    Frequency (f1) of the modal class = 20
    Frequency (f0) of class preceding the modal class = 7
    and Frequency (f2) of the class succeeding the modal class = 5
    Henc, mode of the given data is 32.321.
    Now, substitute the values in the formula of mode, we get
    Mode space equals space l plus space open square brackets fraction numerator straight f subscript 1 minus straight f subscript 0 over denominator 2 straight f subscript 1 minus straight f subscript 0 minus straight f subscript 3 end fraction close square brackets cross times straight h
space space space space space space space equals space 30 plus open square brackets fraction numerator 20 minus 7 over denominator 2 cross times 20 minus 7 minus 5 end fraction close square brackets cross times 5
space space space space space space space equals space 30 plus open parentheses fraction numerator 13 over denominator 40 minus 12 end fraction close parentheses cross times 5
space space space space space space space equals space 30 space plus 65 over 28
space space space space space space space equals space fraction numerator 840 plus 65 over denominator 28 end fraction
space space space space space space space equals space 905 over 28
space space space space space space space equals space 32.321.

    Question 61
    CBSEENMA10009158

    Determine the value of mode from the following frequency distribution table.

    Murks (C.I.)

    No. of students (f)

    0-10

    5

    10-20

    12

    20-30

    14

    30-40

    10

    40-50

    8

    50-60

    6

    Solution

    Here, the maximum class frequency is 14 and the class corresponding to frequency is 20-30.
    So, the modal class is 20-30.
    Thus, we have
    Modal class = 20-30
    l (lower limit of modal class) = 20
    h (class size) = 10
    f1 = (frequency of the modal class) = 14
    f0 = (frequency of class preceding the modal class) = 12
    f2 = (frequency of the class succeeding the modal class) = 10
    Now, substituting these values in the formula of mode, we get
    Mode space equals space l italic space plus space open square brackets fraction numerator straight f subscript 1 minus straight f subscript 0 over denominator 2 straight f subscript 1 minus straight f subscript 0 minus straight f subscript 2 end fraction close square brackets cross times straight h
space space space space space space space space equals space 20 plus open square brackets fraction numerator 14 minus 12 over denominator 2 cross times 14 minus 12 minus 10 end fraction close square brackets cross times 10
space space space space space space space space equals 20 plus open square brackets fraction numerator 2 over denominator 28 minus 22 end fraction close square brackets cross times 10
space space space space space space space space equals 20 plus 2 over 6 cross times 10
space space space space space space space space equals space 20 plus 10 over 3
space space space space space space space space equals space fraction numerator 60 plus 10 over denominator 3 end fraction
space space space space space space space space equals space 70 over 3
space space space space space space space space equals space 23.33 space marks.

    Question 62
    CBSEENMA10009159

    The marks distribution of 30 students in a mathematics are given in the table. Find the mode of this data. Also compare and interpret the mode and mean.

    Class interval

    Number of students (f)

    10-25

    2

    25-40

    3

    40-55

    7

    55-70

    6

    70-85

    6

    85-100

    6

    Solution

    Case I: Finding Mode :
    Here, the maximum class frequency is 7 and the class corresponding to frequency is 40 - 55. So the modal class is 40 - 55.
    Thuse we have
    Model class = 40.55
    = 40
    f1 = 7
    f0 =3
    f= 6
    and h = 15
    Npw substituting these values in the formula of mode, we get
    Mode space equals space l italic space italic plus italic space open square brackets fraction numerator f subscript italic 1 italic minus f subscript italic 0 over denominator italic 2 f subscript italic 1 italic minus f subscript italic 0 italic minus f subscript italic 2 end fraction close square brackets italic cross times h
italic space italic space italic space italic space italic space italic space italic space italic space italic equals italic space italic 40 italic plus open square brackets fraction numerator italic 7 italic minus italic 3 over denominator italic 2 italic cross times italic 7 italic minus italic 3 italic minus italic 6 end fraction close square brackets italic cross times italic 15 italic equals italic 40 italic plus open square brackets fraction numerator italic 4 over denominator italic 14 italic minus italic 9 end fraction close square brackets italic cross times italic 15
italic space italic space italic space italic space italic space italic space italic space italic space italic equals italic space italic 40 italic plus italic 4 over italic 5 italic cross times italic 15 italic equals italic 40 italic plus italic 12 italic equals italic 52

    Case II: Finding Mean

    C.I.

    fi

    xi

    di = x2- 62.5

    fiui = di/h

    fiui

    10-25

    2

    17.5

    - 45

    - 3

    - 6

    25-40

    3

    32.5

    - 30

    - 2

    - 6

    40-55

    7

    47.5

    - 15

    - 1

    - 7

    55-70

    6

    62.5 = A

    0

    0

    0

    70-85

    6

    77.5

    15

    1

    6

    85-100

    6

    92.5

    30

    2

    12

     

    Σfi = 30

         

    Σfiui= -1


    top enclose straight x equals A plus fraction numerator begin display style sum from blank to blank of end style f subscript i u subscript i over denominator begin display style sum from blank to blank of end style f end fraction cross times h
space space equals space 62.5 space plus open parentheses fraction numerator negative 1 over denominator 30 end fraction cross times 15 close parentheses
space equals space 62.5 plus open parentheses fraction numerator negative 1 over denominator 2 end fraction close parentheses
space equals space 62.5 minus 0.5 space equals space 62.

    Hence, the mode of the given data is 52 while median is 62.
    Interpretation : Maximum number of students obtained 52 marks, while on an average student obtained 62 marks.

    Question 63
    CBSEENMA10009160

    The following table shows the marks obtained by 100 students of class X in a school during a particular academic session. Find the mode of this distribution.

    Marks

    No. of students

    Less than 10

    7

    Less than 20

    21

    Less than 30

    34

    Less than 40

    46

    Less than 50

    66

    Less than 60

    77

    Less than 70

    92

    Less than 80

    100

    Solution

    Class Marks (C.I.)

    No. of students (c.f.)

    f

    Less than 10

    7

    7

    Less than 20

    21

    14

    Less than 30

    34

    13

    Less than 40

    46

    12

    Less than 50

    66

    20

    Less than 60

    77

    11

    Less than 70

    92

    15

    Less than 80

    100

    8

    The class (40-50) has maximum frequency i.e. 20 therefore, modal class is 40-50.
                    f = 20
    Now,         l = 40,      h = 10m,   f= 20 and f= 11
                    f= 12  and modal class = 40 - 50 
    therefore space Mode space equals space l space plus space fraction numerator straight f subscript 1 minus straight f subscript 0 over denominator 2 straight f subscript 1 minus straight f subscript 0 minus straight f subscript 2 end fraction cross times straight h
rightwards double arrow space space space Mode space equals space 40 plus fraction numerator 20 minus 12 over denominator 2 left parenthesis 20 right parenthesis minus 12 minus 11 end fraction cross times 10
rightwards double arrow space space Mode space equals space space 40 plus fraction numerator 8 over denominator 40 minus 12 minus 11 end fraction straight x 10
rightwards double arrow space space Mode space equals space 40 plus 8 over 17 cross times 10
rightwards double arrow space Mode space equals space 40 plus 8 over 17
rightwards double arrow space Mode space equals space 40 plus 4.70
rightwards double arrow space Mode space equals space 44.70

    Question 64
    CBSEENMA10009161

    Calculate the median of the following distribution of incomes of employees of a company.

    Income

    No. of Persons

    400-500

    25

    500-600

    69

    600-700

    107

    700-800

    170

    800-900

    201

    900-1000

    142

    1000-1100

    64

    Solution

    Forming the cumulative frquency table, we get

    Income

    No. of Persons

    c.f.

    400-500

    25

    25

    500-600

    69

    94

    600-700

    107

    201

    700-800

    170

    371

    800-900

    201

    572

    900-1000

    142

    714

    1000-1100

    64

    778


    Thus, we have
    Median class = 800 - 900
             l = 8000
      straight n over 2 space space equals space 389
    cf = 371, f = 201 and h = 100
    Now, substituting these values in formula of median, we get
    Median space equals space straight l plus open square brackets fraction numerator begin display style straight n over 2 end style minus cf over denominator straight f end fraction close square brackets cross times straight h
space space space space space space space space space space equals space 800 plus open square brackets fraction numerator 389 minus 371 over denominator 201 end fraction close square brackets cross times 100
space space space space space space space space space space equals space 800 plus fraction numerator 18 cross times 100 over denominator 201 end fraction
space space space space space space space space space space equals space 800 plus 1800 over 201
space space space space space space space space space space equals space 800 plus 8.95 equals 808.95
    Hence, median class Rs. = 808.95

    Question 65
    CBSEENMA10009162

    Find the mean, mode and median for the following data :

    Class

    0-10

    10-20

    20-30

    30-40

    40-50

    Total

    Frequency

    8

    16

    36

    34

    6

    100

    Solution

    I. Finding Mean.
    Wer have A = 25, sum from blank to blank of f subscript 1 equals 100 comma space sum from blank to blank of f subscript i u subscript i equals 14 space a n d space h space equals space 10
    N o w comma space space space space space space top enclose x equals A plus fraction numerator begin display style sum from blank to blank of end style f subscript i u subscript i over denominator begin display style sum from blank to blank of end style f subscript i end fraction cross times h equals 25 plus 14 over 100 cross times 10
             
                 = 25 + 1.4 = 26.4
    II. Finding Meadian
    We have, straight N over 2 equals 50 comma space space c f equals 124 comma space l equals space 20 comma space n equals 10 space a n d space f space equals 36
    Now, 
     Median = l + fraction numerator begin display style straight N over 2 end style minus cf over denominator straight f end fraction cross times h
    equals 20 plus fraction numerator 50 minus 24 over denominator 36 end fraction straight x space 10 space equals space 20 plus 26 over 36 straight x space 10
equals space 20 space plus space 65 over 9 equals fraction numerator 180 plus 65 over denominator 9 end fraction equals 245 over 9 equals 27.2

    III. Finding Mode
    We have,
    Modal class = 20-30 l = 20, h = 10, f1 = 36, f0 = 16 and f2 = 34
    Now,
    Mode space equals space space l plus open square brackets fraction numerator straight f subscript 1 minus straight f subscript 0 over denominator 2 straight f subscript 1 minus straight f subscript 0 minus straight f subscript 2 end fraction close square brackets cross times straight h
space space space space space space space space equals space 20 plus open square brackets fraction numerator 36 minus 16 over denominator 2 cross times 36 minus 16 minus 24 end fraction close square brackets cross times 10
space space space space space space space space equals space 20 plus fraction numerator 20 over denominator 72 minus 50 end fraction cross times 10 equals 20 plus 20 over 22 cross times 10 equals 20 plus 200 over 22
space space space space space space space space equals space 28.8


     
    Question 66
    CBSEENMA10009163

    The median of the following data is 52.5. Find the values of x and y if the total frequency is 100. 

    Class Interval

    Frequency

    0-10

    2

    10-20

    5

    20-30

    x

    30-40

    12

    40-50

    17

    50-60

    20

    60-70

    y

    70-80

    9

    80-90

    7

    90-100

    4

     

    Total 100

    Solution

    C.l.

    fi

    c.f.

    0-10

    2

    2

    10-20

    5

    7

    20-30

    x

    7 + x

    30-40

    12

    19 +

    40-50

    17

    36 + x

    50-60

    20

    56 + x

    60-70

    y

    56 + x + y

    70-80

    9

    65 + x + y

    80-90

    7

    72 + x + y

    90-100

    4

    76 + x + y

    It is given that n = 100
    ∴ 76 + x + y 100 ⇒ x + y = 24 ...(i)
    The median is 52.5 which lies in the class 50-60.
    ∴ l = 50,f = 20, c.f. = 36 + x, h = 10
    Using the formula.
    Median space equals space straight l plus fraction numerator begin display style straight n over 2 end style minus straight c. straight f over denominator straight f end fraction cross times straight h
space space 52.5 equals 50 plus fraction numerator 50 minus 36 minus straight x over denominator 20 end fraction cross times 100
    rightwards double arrow   52.5 - 50 = (14 - x) x 0.5
    rightwards double arrow   2.5 = 7 -0.5x rightwards double arrow25 = 70 - 5x
                    rightwards double arrow 5x = 70 - 25 = 45 
                     rightwards double arrow  x = 9
    Putting this value of x in (i), we get 
         9 + y = 24
    rightwards double arrow  y = 24 - 9 = 15
    Hence,  x = 9 and y= 15

    Question 67
    CBSEENMA10009164

    Calculate the median from the following data:

    Value

    Frequency

    Less than 10

    4

    Less than 20

    16

    Less than 30

    40

    Less than 40

    76

    Less than 50

    96

    Lc9s than 60

    112

    Less than 70

    120

    Less than 80

    125

    Solution

    Forming the Cumulative frequency table we have.

    Value

    Frequency

    C.f

    0-10

    4

    4

    10-20

    12

    16

    20-30

    24

    40

    30-40

    36

    76

    40-50

    20

    96

    50-60

    16

    112

    60-70

    8

    120

    70-80

    5

    125


    Here middle term is open parentheses 125 over 2 close parentheses to the power of th or 62.5th obser vation, which lies in class 30-40.
    Thus,, we have
    Median class = 30 - 40
         l = 30,  straight n over 2 equals space 62.5
    c.f. = 40,   f = 36 and h = 10
    Putting these values in the formula of median
    Median space equals space straight l plus open square brackets fraction numerator begin display style straight n over 2 end style minus straight c. straight f. over denominator straight f end fraction close square brackets cross times straight h
space space space space space space space space space space space equals space 30 plus open square brackets fraction numerator 62.5 minus 40 over denominator 36 end fraction close square brackets cross times 10
space space space space space space space space space space space equals space 30 plus fraction numerator 22.5 over denominator 36 end fraction cross times 10
space space space space space space space space space space
              = 30 + 6.25 = 36.25
    Hence, median = 36.25
    Question 68
    CBSEENMA10009165

    Calculate the median from the following data:

    Size

    Frequency

    More than 50

    0

    More than 40

    40

    More than 30

    98

    More than 20

    123

    More than 10

    165

     

    Solution

    Forming the cumulatiue frequency table, we have

    C.I.

    f

    C.f

    10-20

    42

    42

    20-30

    25

    67

    30-40

    58

    125

    40-50

    40

    165


    Here, middle term is  open parentheses 165 over 2 close parentheses to the power of th or 82.5th obser vation, which lies in class 30-40.
    Thus, we have
    Median class = 30 - 40 l = 30,
    straight n over 2 equals 82.5 comma space straight c. straight f. space equals space 67 comma space straight f space equals space 58 space and space straight h space equals 10
    Putting these values in the formula of median, we get
    Median space equals l plus space open parentheses fraction numerator begin display style straight n over 2 end style minus straight c. straight f. over denominator straight f end fraction close parentheses cross times straight h
space space space space space space space space equals space 30 plus open square brackets fraction numerator 82.5 minus 67 over denominator 58 end fraction close square brackets cross times 10
space space space space space space space space equals space 30 plus fraction numerator 15.5 over denominator 58 end fraction cross times 10
space space space space space space space space space equals space 30 plus fraction numerator 15.5 over denominator 58 end fraction cross times 10
space space space space space space space space space equals space 30 plus 155 over 58
space space space space
            = 3. + 2.67 = 32.67
    Hence, median = 32.67
    Question 69
    CBSEENMA10009166

    The median of the following data is 20.75. Find the missing frequencies x and y, if the total frequency is 100.  

    Class Interval

    0 - 5

    5 - 10

    10 - 15

    15 - 20

    20 - 25

    25 - 30

    30 - 35

    35 - 40

    Frequency

    7

    10

    x

    13

    y

    10

    14

    9

    Solution

    Here, the missing frequencies are x and y.

    Class intervals

    Frequency (f)

    Cumulative frequency

    0-5

    7

    7

    5-10

    10

    17

    10-15

    x

    17 + x

    15-20

    13

    30 + x

    20-25

    y

    30 + x + y

    25-30

    10

    40 + x + y

    30-35

    14

    54 + x + y

    35-40

    9

    63 + x + y

    Total

    100

     

    It is given that N = 100 = Total frequency
    ∴    63 + x + y = 100
    ⇒    x + y = 100 - 63
    ⇒    x + y = 37
    ⇒    y = 37-x    ...(i)
    The median is 20.75 (given), which lies in the class 20-25 ⇒ Median class = 20-25
    Here, we have    l = lower limit of median class = 20, Median class = 20-25
    f = frequency of median class = y
    cf = cumulative frequency of class preceding the median class = 30 + x
    and    h = class size = 5
    Substituting these values in the formula of median, we get
                        
                    space space space space space space space space space space space space space space space space space space space space space space space space space space space space space space space space space space space space space space space space space Median space equals space straight l plus open square brackets fraction numerator begin display style straight N over 2 minus cf end style over denominator straight f end fraction close square brackets cross times straight h
    rightwards double arrow                                            20.75 equals 20 plus open parentheses fraction numerator begin display style 100 over 2 minus left parenthesis 30 plus straight x right parenthesis end style over denominator straight y end fraction close parentheses cross times 5
    rightwards double arrow                                            0.75 space equals space open parentheses fraction numerator 50 minus 30 minus straight x over denominator straight y end fraction close parentheses cross times 5
    rightwards double arrow                                           3 over 4 equals fraction numerator left parenthesis 20 minus x right parenthesis cross times 5 over denominator y end fraction
    rightwards double arrow                                           3y = 400 - 20 x
    rightwards double arrow                                          3(37 - x) = 400 - 20x             [From (i)]
    rightwards double arrow                                          111 - 3x = 400 - 20x
    rightwards double arrow                                           17x = 289
    rightwards double arrow                                            x = 17
    Substituting x = 17 in (i), we get 
                                                        y = 37 -17 = 20
    Hence, the missing frequencies are x = 17 and y = 20.
                     

    Question 70
    CBSEENMA10009167

    A survey regarding the heights (in cm) of 50 girls of class X of a school was conducted and the following data was obtained:

    Height in cm

    120 - 130

    130 - 140

    140 - 150

    150 - 160

    160 - 170

    Total

    Number of girls

    2

    8

    12

    20

    8

    50

    Solution

    Hence we have
    sum from blank to blank of f subscript i equals 50 comma sum from blank to blank of f subscript i u subscript i equals 24 comma space h equals 10 comma space a n d space A space equals space 145
    Now,
    top enclose straight x space equals space A plus fraction numerator begin display style sum from blank to blank of end style f subscript i u subscript i over denominator begin display style sum from blank to blank of end style f subscript i end fraction cross times h equals 145 plus 24 over 50 cross times 10
equals space 145 plus 24 over 5 equals 145 plus 4.8 equals 149.8

    Finding Median:

    Class Interval

    Frequency (f1)

    Cf

    120 - 130

    2

    2

    130 - 140

    8

    10

    140 - 150

    12

    22

    150 - 160

    20

    42

    160 - 170

    8

    50


    Here, Middle term is open parentheses 50 over 2 close parentheses to the power of th space ot space 25 to the power of th observation

    which lies in the class 150-160
    Thus we have
    Median class = 150 - 160
    l = 150, straight n over 2 equals 25, h = 10, cf = 22 and f = 20
    Putting these values in the formula of Median, we get
    Median space equals l italic space plus space open square brackets fraction numerator begin display style straight n over 2 end style minus cf over denominator straight f end fraction close square brackets cross times straight h
equals space 150 plus open square brackets fraction numerator 25 minus 22 over denominator 20 end fraction close square brackets cross times 10 equals 150 plus 3 over 20 cross times 10
equals space 150 plus 3 over 2 equals left parenthesis 150 plus 1.5 right parenthesis space equals 151.5

    III. Finding Mode

    Class Interval

    No. of Students (f)

    120 - 130

    2

    130 - 140

    8

    140 - 150

    12

    150 - 160

    20

    160 - 170

    8

    Here, the man class frequency is 20 and the class corresponding to frequency is 150 - 160, So, the modal class is 150 - 160.
    Thus, we have
    Modal class = 150-160
    l = 150,f1 = 20
    f0 = 12, f2 = 8, n = 10
    Now,
    Substituting these values in the formula of made, we get
    Mode equals l plus open square brackets fraction numerator straight f subscript 1 minus straight f subscript 0 over denominator 2 straight f subscript 1 minus straight f subscript 0 minus straight f subscript 2 end fraction close square brackets cross times straight h
equals space 150 plus open square brackets fraction numerator 20 minus 12 over denominator 2 left parenthesis 20 right parenthesis minus 12 minus 8 end fraction close square brackets cross times 8
equals 150 plus open square brackets fraction numerator 8 over denominator 40 minus 20 end fraction close square brackets cross times 10
equals space 150 plus 4
equals 154


    Question 71
    CBSEENMA10009168

    Find mean, median and mode of the following data:

    Classes

    Frequency

    0-20

    6

    20-40

    8

    40-60

    10

    60-80

    12

    80-100

    6

    100-120

    5

    120-140

    3

    Median space equals space straight l plus open parentheses fraction numerator begin display style straight N over 2 end style minus straight c. straight f. over denominator straight f end fraction close parentheses cross times straight h
equals 60 plus fraction numerator 25 minus 24 over denominator 12 end fraction cross times 20 equals 60 plus 20 over 12
equals space 60 plus 5 over 3 equals 60 plus 1.67 equals 61.67

    Solution

    From the table,   straight n equals sum for blank of straight f subscript straight i equals 50 rightwards double arrow straight n over 2 equals 25 comma space straight a space equals space 70 comma space straight h space equals space 20
    I. Finding Mean:

    Mean space equals space straight a space plus fraction numerator begin display style sum from blank to blank of end style straight u subscript straight i straight f subscript straight i over denominator begin display style sum from blank to blank of end style straight f subscript straight i end fraction cross times straight h equals 70 plus fraction numerator left parenthesis negative 19 right parenthesis over denominator 50 end fraction cross times 20
equals 70 minus 38 over 5 equals 70 minus 7.6 equals 62.4
    Now, 60 - 80 is the class whose cumulative frequency 36 is greater than straight n over 2 equals 25.

    Therefore, 60 - 80 is the median class. Thus, the lower limit (l) of the median class is 60.
    Using the formula for calculating the median.
    II. Finding median:  
    Mode space equals space straight l plus open square brackets fraction numerator straight f subscript straight i straight f subscript 0 over denominator 2 straight f subscript 1 minus straight f subscript 0 minus straight f subscript 2 end fraction close square brackets cross times straight h
space space space space space space space space equals space 60 plus open square brackets fraction numerator 12 minus 10 over denominator 2 cross times 12 minus 10 minus 6 end fraction close square brackets cross times 20
space space space space space space space space equals space 60 equals fraction numerator 2 cross times 20 over denominator 24 minus 16 end fraction equals 60 plus 40 over 8
space space space
            = 60 + 5 = 65

    Question 72
    CBSEENMA10009169

    Find mean, median and mode of the following data:

    Classes

    Frequency

    0 - 50

    2

    50 - 100

    3

    100 - 150

    5

    150 - 200

    6

    200 - 250

    5

    250 - 300

    3

    300 - 350

    1

    Solution


    From the table,   straight n equals space sum from blank to blank of straight f subscript straight i equals 25 rightwards double arrow straight n over 2 equals 12.5 comma space straight a space equals space 175 comma space straight h space equals space 75
|
    I. Finding Mean :

    Mean space equals space straight a space plus fraction numerator begin display style sum from blank to blank of end style straight f subscript straight i straight u subscript straight i over denominator begin display style sum from blank to blank of end style straight f subscript straight i end fraction cross times straight h
space space space space space space space space equals space 175 plus fraction numerator negative 3 over denominator 25 end fraction cross times 50
space space space space space space space space equals space 175 minus 6 equals 169
    Now, 150 - 200 is the class whose cumulative frequency 16 is greater than straight n over 2 equals 12.5. space space  

    Therefore, 150 - 200 is the median class. Thus, the lower limit (l) of the median class is 150.

    II. Finding median:

    Median space equals space straight l plus fraction numerator open parentheses begin display style straight n over 2 end style minus cf close parentheses over denominator straight f end fraction cross times straight h
space space space space space space space space space equals space 150 plus open parentheses fraction numerator 12.5 minus 10 over denominator 6 end fraction close parentheses cross times 50
    III. Finding mode:
    Mode space equals space l italic space plus open square brackets fraction numerator straight f subscript 1 straight f subscript 0 over denominator 2 straight f subscript 1 minus straight f subscript 0 minus straight f subscript 2 end fraction close square brackets cross times straight h
equals space 150 plus open square brackets fraction numerator 6 minus 5 over denominator 2 cross times 6 minus 5 minus 5 end fraction close square brackets cross times 50
equals space 150 equals 1 half cross times 50

    = 150 + 25 = 175

    Question 73
    CBSEENMA10009170

     The following table gives the daily income of 50 workers of a factory :

    Daily income (in Rs.)

    No. of Workers

    100-120

    12

    120-140

    14

    140-160

    8

    160-180

    6

    180-200

    10

    Find the Mean, Mode and Median of the above data.

    Solution

    C.I.

    xi

    fi

    fiixi

    c.f

    100-120

    110

    12

    1320

    12

    120-140

    130

    14

    1820

    26

    140-160

    150

    8

    1200

    34

    160-180

    170

    6

    1020

    40

    180-200

    190

    10

    1900

    50

     

    Σfi = 50

     

    Σfixi = 7260

     

    left parenthesis straight i right parenthesis space Mean space open parentheses top enclose straight x close parentheses equals fraction numerator begin display style sum from blank to blank of end style straight f subscript straight i straight x subscript straight i over denominator begin display style sum from blank to blank of end style straight f subscript straight i end fraction equals 7260 over 50 equals 145.2
    (ii) Here the max. class frequency is 14, and the class corresponding to this frequency is 120-140.
    therefore Model class is 120 - 140, therefore space space straight l space equals space 120 comma space Class size h = 20
    f= freq. of the model class = 14
    f0 = 12,   f= 8
    therefore space space M o d e space equals space l italic plus open square brackets fraction numerator f subscript italic 1 italic minus f subscript italic 0 over denominator italic 2 f subscript italic 1 italic minus f subscript italic 0 italic minus f subscript italic 2 end fraction close square brackets italic cross times h
italic space italic space italic space italic space italic space italic space italic space italic space italic space italic space italic space italic space italic equals italic space italic 120 italic plus fraction numerator open parentheses italic 14 italic minus italic 12 close parentheses italic cross times italic 20 over denominator italic 2 italic cross times italic 14 italic minus italic 12 italic minus italic 8 end fraction
italic space italic space italic space italic space italic space italic space italic space italic space italic space italic space italic space italic space italic equals italic 120 italic plus fraction numerator italic 2 italic cross times italic 20 over denominator italic 8 end fraction italic equals italic 125
italic space
    left parenthesis iii right parenthesis space Here space straight n over 2 equals 25 which lies in the class 120 - 140
    therefore space l = 120. c.f. = 12, f = 14, h = 20
    therefore space thin space M e d i a n space equals space l space plus open square brackets fraction numerator begin display style n over 2 end style minus c. f. over denominator f end fraction close square brackets cross times h
space space space space space space space space space space space space space space equals space 120 plus fraction numerator 13 cross times 20 over denominator 14 end fraction
space space space space space space space space space space space space space space equals 120 plus 130 over 7 equals 120 plus 18.6
space space space space space space space space space space space space space space equals space 138.6
    Hence, mean = 145.2
        mode = 125
      median = 138.6
    Question 74
    CBSEENMA10009171

    Find the mode, median and mean for the following data :

    Marks Obtained

    Number of Students

    25-35

    7

    35-45

    31

    45-55

    33

    55-65

    17

    65-75

    11

    75-85

    1

    Solution

    Marks obtained

    xi

    Frequency fi

    di

    c.f

    fidi

    25-35

    30

    7

    -30

    7

    -210

    35-45

    40

    31

    -20

    38

    -620

    45-55

    50

    33

    -10

    71

    -330

    55-65

    60

    17

    0

    88

    0

    65-75

    70

    11

    10

    99

    110

    75-85

    80

    1

    20

    100

    20

       

    Σf = 100

       

    Σfidi = -1030


    Let A be 60
    Let space straight A space be space 60
Mean space equals space straight a plus fraction numerator begin display style sum from blank to blank of end style straight f subscript straight i straight d subscript straight i over denominator begin display style sum from blank to blank of end style straight f subscript straight i end fraction equals 60 plus open parentheses fraction numerator negative 1030 over denominator 100 end fraction close parentheses
space space space space space space space equals space 60 space minus space 10.3 space equals space 49.7
Modal space class space equals space 45 space minus space 55 comma space straight l space equals space 45
mode space equals space straight l plus open parentheses fraction numerator 33 minus 31 over denominator 66 minus 48 end fraction close parentheses cross times 10 equals 45 plus 2 over 18 cross times 10
space space space space space space space space space equals space 45 space plus space 1.1 space equals space 46.1
Mean space equals space 49.7 comma space space Mode space equals space 46.1
3 space Median space equals space Mode space plus space 2 space Mean
Median space equals space fraction numerator 46.1 plus 99.4 over denominator 3 end fraction
Median space equals space 48.5
    Question 75
    CBSEENMA10009172

    The table given below shows the frequency distribution of the scores obtained by 200 candidates in a BCA examination.

    Score

    No. of candidates

    200-250

    30

    250-300

    15

    300-350

    45

    350-400

    20

    400-450

    25

    450-500

    40

    500-550

    10

    550-600

    15

    Draw cumulative frequency curves by using (i) 'less than series', (ii) 'more than series'.

    Solution

    e assume a class interval 150-200 prior to the first class interval 200-250 wtih zero frequency.
    Cumulative frequency distribution [Less than Series]

    Scroe

    c.f.

    Less than 200

    0

    Less than 250

    30

    Less than 300

    45

    Less than 350

    90

    Less than 400

    110

    Less than 450

    135

    Less than 500

    175

    Less than 550

    185

    Less than 600

    200 


    Now, we plot the points : (200, 0), (250, 30), (300, 45), (350, 90), (400, 110), (450, 135), (500, 175), (550, 185), (600, 200).

    More than Series

    Scroe

    c.f.

    More than 200

    200

    More than 250

    170

    More than 300

    155

    More than 350

    110

    More than 400

    90

    More than 450

    65

    More than 500

    25

    More than 550

    15

    More than 600

    0

    Now, we plot the pts.: (200, 200), (250,170), (300,155), (350,110), (400, 90), (450, 65), (500, 25), (550, 15) and (600, 0).

    Question 76
    CBSEENMA10009173

    Draw both types of cumulative freqneucy curve on the same graph paper and then determine the median.

    Marks obtained

    No. of students

    50-60

    4

    60-70

    8

    70-80

    12

    80-90

    6

    90-100

    6

    Solution

    c.f. distribution table :

    Marks

    No. of students

    c.f. (Less than)

    c.f (More tlian)

           

    50-60

    4

    4

    36

    60-70

    8

    12

    32

    70-80

    12

    24

    24

    80-90

    6

    30

    12

    90-100

    6

    36

    6

    Now, we plot the points (60,4), (70,12), (80,24), (90,30), (100,36), for less than series.
    And. (50,36), (60,32), (70,24), (80,12), (90,6) for more than series.
    The two curves drawn intersect each other at point, say P. Through this point P, draw a vertical line, which meets x-axis at 76.
    So, median = 76

    Question 77
    CBSEENMA10009174

    Draw 'less than' and 'more than' ogive curve from the following and indicate the value of median.

    Marks

    No. of students (f)

    0-5

    7

    5-10

    10

    10-15

    20

    15-20

    13

    20-25

    12

    25-30

    10

    30-35

    14

    35-40

    9

    Solution

     c.f. distribution

    Marks

    No. of students

    c.f. (Less than)

    c.f (More than)

    0-5

    7

    7

    95

    5-10

    10

    17

    88

    10-15

    20

    37

    78

    15-20

    13

    50

    58

    20-25

    12

    62

    45

    25-30

    10

    72

    33

    30-35

    14

    86

    23

    35-40

    9

    95

    9

    Now we plot the points (5,7), (10,17), (15,37), (20, 50), (25, 62), (30, 72), (35, 86), (40, 95) for less than series. And (0,95), (5,88), (10,78), (15,58), (20, 45), (25,33), (30,23), (35,9) for more than series.

    The two curves drawn intersect each other at P. Through this P, draw a vertical line, which meets the x-axis at 20. So median = 20.

    Question 78
    CBSEENMA10009175

    The following table gives the daily income of 50 workers of a factory. Draw both types (less than type and greater than type) ogives and determine the median of the data.

    Daily Income (in Rs.)

    Number of Workers

    100-120

    12

    120-140

    14

    140-160

    8

    160-180

    6

    180-200

    10

    Solution

    Marks

    No of students

    c.f. (Less than)

    c.f. (More than)

    100-120

    12

    12

    50

    120-140

    14

    26

    38

    140-160

    8

    34

    24

    160-180

    6

    40

    16

    180-200

    10

    50

    10

    No we plot the points (100, 0), (120, 12), (140, 26), (160, 34), (180, 40), (200, 50) for less than series. And (100, 50), (120, 38), (140, 24), (160,16), (180,10) for more than series.


    The two curves drawn intersect each other at P. Through this P, draw a vertical line, which meets the x-axis at 140. So median = 140.
    Question 79
    CBSEENMA10009176

    During the medical check-up of 35 students of a class their weights were recorded as follows:

    Weight (in kg.)

    No. of Students

    38—40

    3

    40—42

    2

    42—44

    4

    44—46

    5

    46—48

    14

    48—50

    4

    50—52

    3

    Draw a less than type and a more than type ogive from the given data. Hence obtain the median weight from the graph.

    Solution

    Classes

    f

    c.f.

     

    c.f.

     

    38-40

    3

    3

    (40, 3)

    35

    (38, 35)

    40-42

    2

    5

    (42, 5)

    32

    (40, 32)

    42-44

    4

    9

    (44, 9)

    30

    (42, 30)

    44-46

    5

    14

    (46, 14)

    26

    (44. 26)

    46-48

    14

    28

    (48, 28)

    21

    (46, 21)

    48-50

    4

    32

    (50, 32)

    7

    (48, 7)

    50-52

    3

    35

    (52, 35)

    3

    (50, 3)

    We plot the points (40, 3), (42, 5), (44, 9), (46, 14), (48, 28), (50, 32), (52, 35). We join these points with a smooth curve to get the 'less than' ogive as shown in Fig. 14.20.

    Then, we plot the points (38, 35), (40, 32), (42, 30), (44, 26), (46, 21), (48, 7), (50, 3) on the same axes. By joining these points with a smooth curve to get 'more than' ogive. Since, the two curves intersect at the point, whose abscissa is 47 (approx). Hence, the required median weight is 47 kg (approx.).

    Question 85
    CBSEENMA10009182
    Question 130
    CBSEENMA10009227
    Question 132
    CBSEENMA10009229

    Find the median of the following data :
    31, 15, 47, 91, 12, 56, 29, 61, 65, 55.

    Solution

    Solution not provided.
    Ans.  51

    Question 133
    CBSEENMA10009230

    Find the mean of the first six prime numbers.

    Solution

    Solution not provided.
    Ans.  6.83

    Question 135
    CBSEENMA10009232
    Question 136
    CBSEENMA10009233
    Question 139
    CBSEENMA10009236
    Question 140
    CBSEENMA10009237
    Question 151
    CBSEENMA10009248
    Question 152
    CBSEENMA10009249
    Question 166
    CBSEENMA10009263
    Question 170
    CBSEENMA10009267

    Find the mean, mode and median of the following data :

    Classes

    Frequency

    0-10

    3

    10-20

    8

    20-30

    10

    30-40

    15

    40-50

    7

    50-60

    4

    60-70

    3

    Solution
    Solution not provided
    Ans.  Mean : 32.8, Mode : 33.85, Median= 32.67
    Question 171
    CBSEENMA10009268

     Find the mean, mode and median of the following data:

    Classes

    Frequency

    0-10

    3

    10-20

    4

    20-30

    7

    30-40

    15

    40-50

    10

    50-60

    7

    60-70

    4

    Solution
    Solution not provided
    Ans.  Mean : 37.4, Median : 36.15, Mode : 37.33 
    Question 172
    CBSEENMA10009269
    Question 173
    CBSEENMA10009270

    Find the mean, mode and median of the following data:

    Classes

    Frequency

    0-10

    5

    10-20

    8

    20-30

    15

    30-40

    20

    40-50

    14

    50-60

    8

    60-70

    5

    Solution
    Solution not provided
    Ans. Mean : 34.867, Median : 34.75, Mode : 34.54 
    Question 174
    CBSEENMA10009271
    Question 175
    CBSEENMA10009272

    Find the mean, mode and median of the following data:

    Classes

    Frequency

    0-10

    6

    10-20

    8

    20-30

    10

    30-40

    15

    40-50

    5

    50-60

    4

    60-70

    2

    Solution

    Solution not provided.
    Ans.   Mean : 30, Median : 30.67, Mode :33.33

    Question 192
    CBSEENMA10009293

    Why is tossing a coin considered to be a fairway of deciding, which team should get the ball at the beginning of a football game?

    Solution
    When we toss a coin, the outcomes are equally likely. So, the result of an individual coin toss is completely unpredictable.
    Question 194
    CBSEENMA10009295

    If P(E) = 0.05, what is the probability of ‘not E’?

    Solution

    It is given that :
    P(E) = 0.05, then Probability of ‘not E’ = 1 – P(E)
    ⇒    P(E) = 1 – 0.05 = 0.95.

    Question 195
    CBSEENMA10009296

    A bag contains lemon flavoured candies only. Malini takes out one candy without looking into the bag. What is the probability that she takes out :
    (i) an orange flavoured candy?
    (ii) a lemon flavoured candy?

    Solution

    (i) 0, because the bag contains the lemon flavoured candies only.
    (ii) 1, because the bag contains lemon flavoured candies only.

    Question 196
    CBSEENMA10009297

    It is given that the probability of 2 students in a group of 3 students, not having the same birthday is 0.992. What is the probability that the 2 students have the same birthday?

    Solution

    Probability that the 2 students have the same birthday
    = 1 – Probability that the 2 students have not the same birthday
    = 1 – 0.992 = 0.008.

    Question 197
    CBSEENMA10009298

    A bag contains 3 red balls and 5 black balls. A ball is drawn at random from the bag. What is the probability that the ball drawn is (i) red? (ii) not red?

    Solution

    Number of red balls in the bag = 3
    Number of black balls in the bag = 5
    Total number of balls in the bag = 3 + 5 = 8
    i.e.    n(s) = 8
    (i) Let A be the favourable outcomes of getting red balls, then
    n(A) = 3
    Therefore, P(A) equals fraction numerator straight n left parenthesis straight A right parenthesis over denominator straight n left parenthesis straight A right parenthesis end fraction equals 3 over 8
    Let Be be the favourable outcomes of getting ‘not red'balls. Then,
             n(B) = 5
    Therefore, P(B) = fraction numerator straight n left parenthesis straight B right parenthesis over denominator straight n left parenthesis straight S right parenthesis end fraction equals 5 over 8.

    Question 198
    CBSEENMA10009299

    A box contains 5 red marbles, 8 white marbles and 4 green marbles. One marble is taken out of the box at random. What is the probability that the marble taken out will be (i) red? (ii) white? (iii) not green?

    Solution

    Number of red marbles = 5
    Number of white marbles = 8
    Number of green marbles = 4
    Total number of marbles = 5 + 8 + 4 = 17
    i.e.    n(s) = 17
    (i) Let A be the favourable outcomes of getting red marble, then
                n(A) = 5
    Therefore, P(A) = fraction numerator straight n left parenthesis straight A right parenthesis over denominator straight n left parenthesis straight s right parenthesis end fraction equals 5 over 17
    (ii) Let B be the favourable outcomes of getting white marbles, then n(B) = 8
    Therefore, P(B) = fraction numerator straight n left parenthesis straight B right parenthesis over denominator straight n left parenthesis straight s right parenthesis end fraction equals 5 over 17
    (iii) Let C be the favourable outcomes of getting ‘not green’ marble, then
                     n(C) = 5 + 8 = 13
    Therefore, P(C) = fraction numerator straight n left parenthesis straight C right parenthesis over denominator straight n left parenthesis straight s right parenthesis end fraction equals 13 over 17

    Question 199
    CBSEENMA10009300

    A piggy bank contains hundred 50 p coins, fifty Re. 1 coins, twenty Rs. 2 coins and ten Rs. 5 coins. If it is equally likely that one of the coins will fall out when the bank is turned upside down, what is the probability that the coin (i) will be a 50 p coin ? (ii) will not be a Rs. 5 coin?

    Solution

    Number of 50 p coins = 100
    Number of Re. 1 coins = 50
    Number of Rs. 2 coins = 20
    Number of Rs. 5 coins = 10
    Total number of coins = 180
    i.e.    n(s) = 180
    (i) Let A be the favourable outcomes of getting a 50 p coin, then
             
                                 n(A) = 100
     Therefore,  P(A) = fraction numerator straight n left parenthesis straight A right parenthesis over denominator straight n left parenthesis straight s right parenthesis end fraction equals 100 over 180 equals 5 over 9
    (ii) Let B be the favourable outcomes of getting ‘not a Rs. 5 coin’. Then
                              n(B) = 170
     Therefore, P(B) = fraction numerator straight n left parenthesis straight B right parenthesis over denominator straight n left parenthesis straight s right parenthesis end fraction equals 170 over 180 equals 17 over 18

    Question 200
    CBSEENMA10009301

    Gopibuys a fish from a shop for his aquarium. The shopkeeper takes out one fish at random from a tank containing 5 male fish and 8 female fish.
    What is the probability that the fish taken out is a male fish?

    Solution

    Number of male fish = 5
    Number of female fish = 8
    Total number of fish = 13
    i.e.    n(s) = 13
    Let A be the favourable outcomes of getting a male fish. Then
           
                     N(a) = 5
    Therefore, P(A) = fraction numerator straight n left parenthesis straight A right parenthesis over denominator straight n left parenthesis straight s right parenthesis end fraction equals 5 over 13

    Question 201
    CBSEENMA10009302

    A game of chance consists of spinning an arrow which comes to rest point at one of the numbers 1, 2, 3, 4, 5, 6, 7, 8 and these are equally likely outcomes. What is the probability that it will point at


    (i)    8? 
    (ii)    an odd number?
    (iii)    a number greater than 2?
    (iv)    a number less than 9?

    Solution

    Total number of possible outcomes in the game = 8
    i.e.    n(s) = 8
    (i) Let A be the favourable outcomes of getting 8. Then
                  n(A) = 1
    Therefore, P(A) = fraction numerator straight n left parenthesis straight A right parenthesis over denominator straight n left parenthesis straight s right parenthesis end fraction equals 1 over 8
    (ii) Let B be the favourable outcomes of getting an odd number. Then
                       B = {1, 3, 5, 7}
    i.e.              n(B) = 4
    Therefore, P(B) = fraction numerator straight n left parenthesis straight B right parenthesis over denominator straight n left parenthesis straight s right parenthesis end fraction equals y over 8 equals 1 half
    (iii) Let C be the favourable outcomes of getting a number greater than 2. Then,
                   C = {3, 4, 5, 6, 7, 8}
    i.e.           n(C) = 6
    Therefore, P(C) = fraction numerator straight n left parenthesis straight C right parenthesis over denominator straight n left parenthesis straight s right parenthesis end fraction equals 6 over 8 equals 3 over 4
    (iv) Let D be the favourable outcomes of getting a number less than 9. Then,
                   D = {1, 2, 3, 4, 5, 6, 7, 8}
    i.e.           n(D) = 8
    Therefore, P(D) = fraction numerator n left parenthesis D right parenthesis over denominator n left parenthesis s right parenthesis end fraction equals 8 over 8 equals 1

    Question 202
    CBSEENMA10009303

    A die is thrown once, find the probability of getting
    (i)    a prime number.
    (ii)    a number lying between 2 and 6.
    (iii)    an odd number.

    Solution

    If we throw a die once, then possible outcomes (s) are
    S = {1, 2, 3, 4, 5, 6}
    ⇒    n( S) = 6
    (i) Let E be the favourable outcomes of getting a prime number then
        
                         E = {2, 3, 5}
    rightwards double arrow space             n(E) = 3
    Therefore,    P(E) = fraction numerator straight n left parenthesis straight E right parenthesis over denominator straight n left parenthesis straight S right parenthesis end fraction equals 3 over 6 equals 1 half
    (ii) Let F be the favourable outcomes of getting a number lying between 2 and 6, then
                              F = {3, 4, 5}
    rightwards double arrow                   n(F) = 3
    Therefore,   P(F) = fraction numerator straight n left parenthesis straight F right parenthesis over denominator straight n left parenthesis straight S right parenthesis end fraction equals 3 over 6 equals 1 half
    (iii) Let G be the favourable outcomes of getting an odd number, then
                     
                           G = {1, 3, 5}
    rightwards double arrow               n(G) = 3
    Therefore,     P(G) = fraction numerator straight n left parenthesis straight G right parenthesis over denominator straight n left parenthesis straight S right parenthesis end fraction equals 3 over 6 equals 1 half

    Question 203
    CBSEENMA10009304

    One card is drawn from a well-shuffled deck of 52 cards. Find the probability of getting

    (i)    a king of red colour.

    Solution

    Let E be the favourable outcomes of getting a king of red colour, then n(E) = 2

    Therefore,
    straight P left parenthesis straight E right parenthesis space equals space fraction numerator straight n left parenthesis straight E right parenthesis over denominator straight n left parenthesis straight S right parenthesis end fraction equals 2 over 52 equals 1 over 26

    Question 204
    CBSEENMA10009305

    One card is drawn from a well-shuffled deck of 52 cards. Find the probability of getting

    a face card.

    Solution

    Let F be the favourable outcomes of getting a face card, then

    n(F) = 12

    Therefore,
    P left parenthesis F right parenthesis space equals fraction numerator n left parenthesis F right parenthesis over denominator n left parenthesis S right parenthesis end fraction equals 12 over 52 equals 3 over 13

    Question 205
    CBSEENMA10009306

    One card is drawn from a well-shuffled deck of 52 cards. Find the probability of getting

    a red face card.

    Solution

    Let G be the favourable outcomes of a red face card, then

    n(G) = 6

    Therefore,

     straight P left parenthesis straight G right parenthesis space equals fraction numerator straight n left parenthesis straight G right parenthesis over denominator straight n left parenthesis straight S right parenthesis end fraction equals 6 over 52 equals 3 over 26
    Question 206
    CBSEENMA10009307

    One card is drawn from a well-shuffled deck of 52 cards. Find the probability of getting 
    the Jack of hearts. 

    Solution

    Let H be the favourable outcomes of getting the jacks from the hearts, then n(H) = 1

    Therefore,
    straight P left parenthesis straight H right parenthesis space equals fraction numerator straight n left parenthesis straight H right parenthesis over denominator straight n left parenthesis straight S right parenthesis end fraction equals 1 over 52

    Question 207
    CBSEENMA10009308

    One card is drawn from a well-shuffled deck of 52 cards. Find the probability of getting 
     a spade

    Solution

    Let I be the favourable outcomes of getting a spade, then
    n(I) = 13
    Therefore :    
    P left parenthesis I right parenthesis space equals fraction numerator n left parenthesis I right parenthesis over denominator n left parenthesis S right parenthesis end fraction equals 13 over 52 equals 1 fourth.

    Question 208
    CBSEENMA10009309

    One card is drawn from a well-shuffled deck of 52 cards. Find the probability of getting 
    the queen of diamonds.

    Solution

    Let J be the favourable outcomes of getting thez queen of diamonds, then n(J) = 1

    Therefore,
    straight P left parenthesis straight J right parenthesis space equals fraction numerator straight n left parenthesis straight J right parenthesis over denominator straight n left parenthesis straight S right parenthesis end fraction equals 1 over 52

    Question 210
    CBSEENMA10009311

    Five cards - the ten, jack, queen, king and ace of diamonds are well-shuffled with their face down wards. One card is then picked up at random.

    (ii) If the queen is drawn and put aside, what is the probability that the second card picked up is (a) an ace (b) a queen.

    Solution

    If the Queen is drawn and put aside then possible outcomes become 4
    i.e.,    n(S) = 4
    (a) Let A be the favourable outcomes that the picked up card is an ace, then
    n(A) = {1}
    Therefore,
    straight P left parenthesis straight A right parenthesis equals fraction numerator straight n left parenthesis straight A right parenthesis over denominator straight n left parenthesis straight S right parenthesis end fraction equals 1 fourth

    (b) Let B be the favourable outcomes that the picked up card is a queen, then n(B) = 0

    Therefore,
    straight P left parenthesis straight B right parenthesis equals fraction numerator straight n left parenthesis straight B right parenthesis over denominator straight n left parenthesis straight S right parenthesis end fraction equals 0 over 4 equals 0

    Question 211
    CBSEENMA10009312

    12 defective pens are accidentally mixed with 132 good ones. It is not possible to just look at a pen and tell whether or not it is defective. One pen is taken out at random from this lot. Determine the probability that the pen take out is a good one.

    Solution

    Number of defective pens = 12
    Number of non-defective pens = 132
    Total number of pens = 12 + 132 = 144
    Let ‘A’ be the favourable outcomes of getting the pen taken out is a good one. Then, n(A) = 132
    Therefore comma space space straight P left parenthesis straight A right parenthesis space equals space fraction numerator straight n left parenthesis straight A right parenthesis over denominator straight n left parenthesis straight S right parenthesis end fraction equals 132 over 144 equals 11 over 12.

    Question 212
    CBSEENMA10009313

    (i) A lot of 20 bulbs contain 4 defectuve ones. One bulb is drawn at random from the lot. What is the probability that this bulb is defective ?

    (ii) Suppose the bulb drawn in (i) is not defective and is not replaced.
    Now one bulb is drawn at random from the rest. What is the probability that this bulb is not defective ?

    Solution

    (i) Total number of bulbs = 20
    i.e.    n(S) = 20
    Let A be the favourable outcomes of getting defective bulbs. Then, n(A) = 4
    Therefore, P(A) = fraction numerator n left parenthesis A right parenthesis over denominator n left parenthesis S right parenthesis end fraction equals 4 over 20 equals 1 fifth
    (ii) Total number of bulbs = 19 Let B be the favourable outcomes of getting non defective bulbs. Then  n(B) = 19 - 4 = 15
    Therefore, P(B) = fraction numerator n left parenthesis B right parenthesis over denominator n left parenthesis S right parenthesis end fraction equals 15 over 19.

    Question 213
    CBSEENMA10009314

    A box contains 90 discs which are numbered from 1 to 90. If one disc is drawn at random from the box, find the probability that it bears (i) a two-digit number, (ii) a perfect square number, (iii) a number divisible by 5.

    Solution

    Total number of discs in the box = 90 i.e.    n(S) = 90
    (i) Let A be favourable outcomes of getting a two digit number. Then n(A) = 81
    Therefore,  P(A) = fraction numerator straight n left parenthesis straight A right parenthesis over denominator straight n left parenthesis straight S right parenthesis end fraction equals 81 over 90 equals 9 over 10

    (ii) Let B be the favourable outcomes of getting a perfect square number. Then
    B = {1, 4, 9, 16, 25, 36, 49, 64, 81} i.e.    n(B) = 9
    Therefore, P(B) = fraction numerator straight n left parenthesis straight B right parenthesis over denominator straight n left parenthesis straight S right parenthesis end fraction equals 9 over 90 equals 1 over 10.
    (iii) Let C be the favourable outcomes of setting a number divisible by 5.
    Then,     n(C) = 18
    Therefore, P(C) = fraction numerator straight n left parenthesis straight C right parenthesis over denominator straight n left parenthesis straight S right parenthesis end fraction equals 18 over 90 equals 1 fifth.

    Question 214
    CBSEENMA10009315

    A child has a die whose six faces show the letters as given below :

    box enclose bold A bold space box enclose bold B bold space box enclose bold C bold space box enclose bold D bold space box enclose bold E bold space box enclose bold A

    The die is thrown once. What is the probability of getting (i) A? (ii) D?

    Solution

    Total number of faces in a die = 6 i.e.    n(S) = 6
    (i) Let E be the favourable outcomes of setting A. Then,

    n(E) = 2
    Therefore,  P(E) = fraction numerator straight n left parenthesis straight E right parenthesis over denominator straight n left parenthesis straight S right parenthesis end fraction equals 2 over 6 equals 1 third.

    (ii) Let A be the favourable outcomes of setting D. Then
    n(D) = 1
    Therefore, P(D) = fraction numerator straight n left parenthesis straight D right parenthesis over denominator straight n left parenthesis straight S right parenthesis end fraction equals 1 over 6.

    Question 215
    CBSEENMA10009316

    Suppose you drop a die at random on the rectangular region show in Fig. 15.6. What is the probability that it will tend inside the circle with diameter 1 m?



    Solution
    Area of rectangular region = 3 x 2 = 6 m2 Diameter of the circle = 1 m
    therefore     Radius = 1 half space m
    So,   Area = πr squared equals straight pi open parentheses 1 half close parentheses squared equals straight pi over 4 straight m squared
    Now, Probability that the die will land inside
    the circle = fraction numerator begin display style straight pi over 4 end style over denominator 6 end fraction equals straight pi over 24.

    Question 216
    CBSEENMA10009317

    A lot consists of 144 ball pens of which 20 are defective and the others are good. Nuri will buy a pen if it is good, but will not buy if it is defective. The shopkeeper draws one pen at random and gives it to her. What is the probability that
    (i)    She will buy it?
    (ii)    She will not buy it?

    Solution

    Total number of ball pens = 144 i.e.    n( S) = 144
    (i) Let A be the favourable outcomes of buying a ball pen by her. Then
    n( A) = 144 – 20 = 124
    Therefore, P(A) = fraction numerator straight n left parenthesis straight A right parenthesis over denominator straight n left parenthesis straight S right parenthesis end fraction equals 124 over 144 equals 31 over 36

    (ii) Let B be the favourable outcomes of not buying a ball pen. Then,
    P(B) = 1 – P(A)
    equals 1 minus 31 over 36 equals 5 over 36

    Question 217
    CBSEENMA10009318

    (i) Complete the following table :

    (ii) A student argues that ‘there are 11 possible outcomes 2, 3, 4, 5, 6, 7, 8, 9, 10, 11 and 12.

    Therefore, each of them has a probability 1 over 11Do you agree with this argument ? Justify your answer.

    Solution

    Total number of possible outcomes = 36 i.e.    n( S) = 36
    (i) Let A be the favourable outcomes of getting the sum as 2. Then
    A = (1,1) i.e.    n(A) = 1
    Therefore, P(A) = fraction numerator straight n left parenthesis straight A right parenthesis over denominator straight n left parenthesis straight S right parenthesis end fraction equals 1 over 36.

    (ii) Let B be the favourable outcomes of getting the sum as 3. Then

    B = (1,2), (2,1) i.e.    n(B) = 2
    Therefore,  P(B) = fraction numerator straight n left parenthesis straight b right parenthesis over denominator straight n left parenthesis straight S right parenthesis end fraction equals 2 over 36 equals 1 over 18

    (iii) Let C be the favourable outcomes of getting the sum as 4. Then

    C = (2,2), (1,3), (3,1) i.e.    n(C) = 3
    Therefore, P(C) = fraction numerator straight n left parenthesis straight C right parenthesis over denominator straight n left parenthesis straight S right parenthesis end fraction equals 3 over 36 equals 1 over 12

    (iv)    Let D be the favourable outcomes of getting the sum as 5. Then
    D = {(1,4), (4,1), (2,3), (3,2)}.
    (v)    Let E be the favourable outcomes of getting the sum as 6. Then
    E = {(1,5), (5,1), (2,4), (4,2), (3,3)} i.e.    n(E) = 5
    Therefore, P(E) = fraction numerator straight n left parenthesis straight E right parenthesis over denominator straight n left parenthesis straight S right parenthesis end fraction equals 5 over 36

    (iv)    Let D be the favourable outcomes of getting the sum as 5. Then

    D = {(1,4), (4,1), (2,3), (3,2)}.

    (v)    Let E be the favourable outcomes of getting the sum as 6. Then

    E = {(1,5), (5,1), (2,4), (4,2), (3,3)} i.e.    n(E) = 5
    Therefore, P(F) = fraction numerator straight n left parenthesis straight F right parenthesis over denominator straight n left parenthesis straight S right parenthesis end fraction equals 6 over 36 equals 1 over 6

    (vii) Let G be the favourable outcomes of getting the sum as 8. Then

    G = (2,6), (6,2), (3,5), (5,3), (4,4) i.e.    n(G) = 5
    Therefore, P(G) = fraction numerator straight n left parenthesis straight G right parenthesis over denominator straight n left parenthesis straight S right parenthesis end fraction equals 5 over 36

    (viii) Let H be the favourable outcomes of getting the sum as 9. Then

    H = (3,6), (6,3), (4,5), (5,4) i.e.    n(H) = 4
    Therefore , P(H) = fraction numerator straight n left parenthesis straight H right parenthesis over denominator straight n left parenthesis straight S right parenthesis end fraction equals 4 over 36 equals 1 over 9.

    (ix) Let I be the favourable outcomes of getting the sum as 10. Then
    1 = (4,6), (6,4), (5,5)
    Le.    n(1) = 3
    Therefore, P(I) = fraction numerator straight n left parenthesis straight I right parenthesis over denominator straight n left parenthesis straight S right parenthesis end fraction equals 3 over 36 equals 1 over 12.

    (x) Let J be the favourable outcomes of getting the sum as 11. Then

    J = (6,5), (5,6) i.e.    n(J) = 2
    Therefore, P(J) = fraction numerator straight n left parenthesis straight J right parenthesis over denominator straight n left parenthesis straight S right parenthesis end fraction equals 2 over 36 equals 1 over 18.

    (xi) Let K be the favourable outcomes of getting the sum as 12. Then
    K = (6,6) i.e.    n(K) = 1
    Therefore, P(K) = fraction numerator straight n left parenthesis straight K right parenthesis over denominator straight n left parenthesis straight S right parenthesis end fraction equals 1 over 36

    Question 218
    CBSEENMA10009319

    A game consists of tossing a one rupee coin 3 times and noting its outcome each time. Hanif wins if all the tosses give the same result i.e., three heads or three tails and toses otherwise. Calculate the probability that Hanif will lose the game.

    Solution

    If we toss a one rupee coin then possible outcomes are
    S= {HHH,TTT, HHT, HTH, HTT,THH,THT,TTH}
    i.e.    n(S) = 8
    Let A be the favourable outcomes of losing the game. Then
    A = {HHT, HTH, HTT,THH,THT,TTH}
    i.e.,    (A) = 6
    Therefore, P(A) = fraction numerator straight n left parenthesis straight A right parenthesis over denominator straight n left parenthesis straight S right parenthesis end fraction equals 6 over 8 equals 3 over 4

    Question 219
    CBSEENMA10009320

    A die is thrown twice. What is the probability that
    (i) 5 will not Come up either time?
    (ii) 5 will come up at least once?

    Solution
    When a die is thrown twice, then possible outcomes are

    (0 Let A be the favourable outcomes that 5 will not come up either time.

    (ii) Let B be the favourable outcomes that 5 will come at least once. Then
    Question 220
    CBSEENMA10009321

    Which of the following arguments are correct and which are not correct ? Give reasons for your answer.

    (i) If two coins are tossed simultaneously there are three possible outcomes—two heads, two tails or one of each. Therefore, for each of these outcomes, the probability is 1 third.

    Solution

    (i) If two coins are tossed at the same time the possible outcomes are :

    S = (H,H), (H,T), (T,H), (T,T) i.e.    n(S) = 4

    So, the probability of each occurrence  = 1 fourth
    Thus, the given statement is wrong.

    Question 221
    CBSEENMA10009655

    A number is chosen at random from the number –3, –2, –1, 0, 1, 2, 3. What will be the probability that square of this number is less then or equal to 1? 

    Solution

    S = {−3, −2, −1, 0, 1, 2, 3}
    Let E be the event of getting a number whose square is less than or equal to 1.
    So, E = {−1, 1, 0}
    P(E)=3/7.
    Hence, the probability of getting a number whose square is less than or equal to is 3/7.

    Question 222
    CBSEENMA10009663

    Two different dice are thrown together. Find the probability that the numbers obtained
    (i) have a sum less than 7
    (ii) have a product less than 16
    (iii) is a doublet of odd numbers.

    Solution

    The outcomes when two dice are thrown together are
    (1,1), (1,2), (1,3), (1,4), (1,5), (1,6)
    (2,1), (2,2), (2,3), (2,4), (2,5), (2,6)
    (3,1), (3,2), (3,3), (3,4), (3,5), (3,6)
    (4,1), (4,2), (4,3), (4,4), (4,5), (4,6)
    (5,1), (5,2), (5,3), (5,4), (5,5), (5,6)
    (6,1), (6,2), (6,3), (6,4), (6,5), (6,6)
    Total number of outcomes = 36

    (i) Let A be the event of getting the numbers whose sum is less than 7.

    The outcomes in favour of event A are (1, 1), (1,2), (1,3), (1,4), (1,5), (2,1), (2,2), (2,3), (2,4), (3,1), (3,2), (3,3), (4,1), (4,2) and (5,1).
    Number of favourable outcomes = 15
    therefore space straight P left parenthesis straight A right parenthesis space equals space fraction numerator Number space of space favourable space outcomes over denominator Total space number space of space outcomes end fraction space equals space 15 over 36 space equals space 5 over 12

    (ii) Let B be the event of getting the numbers whose product is less than 16.

    The outcomes in favour of event B are (1,1), (1,2), (1,3), (1,4), (1,5), (1,6), (2,1), (2,2), (2,3), (2,4), (2,5), (2,6), (3,1), (3,2), (3,3), (3,4), (3,5), (4,1), (4,2), (4,3), (5,1), (5,2), (5,3), (6,1) and (6,2).

    Number of favourable outcomes = 25
    therefore space straight P left parenthesis straight A right parenthesis space equals space fraction numerator Number space of space favourable space outcomes over denominator Total space number space of space outcomes end fraction space equals space 25 over 36 space

    iii) Let C be the event of getting the numbers which are doublets of odd numbers.

    The outcomes in favour of event C are (1,1), (3,3) and (5,5).

    Number of favourable outcomes = 3
    therefore space straight P left parenthesis straight A right parenthesis space equals space fraction numerator Number space of space favourable space outcomes over denominator Total space number space of space outcomes end fraction space equals space 3 over 36 space equals space 1 over 12 space

    Question 223
    CBSEENMA10009669

    Peter throws two different dice together and finds the product of the two numbers obtained. Rina throws a die and squares the number obtained. Who has the better chance to get the number 25. 

    Solution

    Let us first write the all possible oucomes when Peter throws two different dice together.

    (1,1), (1,2), (1,3), (1,4), (1,5), (1,6)
    (2,1), (2,2), (2,3), (2,4), (2,5), (2,6)
    (3,1), (3,2), (3,3), (3,4), (3,5), (3,6)
    (4,1), (4,2), (4,3), (4,4), (4,5), (4,6)
    (5,1), (5,2), (5,3), (5,4), (5,5), (5,6)
    (6,1), (6,2), (6,3), (6,4), (6,5), (6,6)

    ∴ Total number of outcomes = 36

    The favorable outcome for getting the product of numbers on the dice equal to 25 is (5, 5).

    Favourable number of outcomes = 1

    ∴ Probability that Peter gets the product of numbers as 25
    = fraction numerator Favourable space number space of space outcomes over denominator Total space number space of space outcomes end fraction space equals space 1 over 36

    The outcomes when Rina throws a die are 1, 2, 3, 4, 5, 6.

    ∴ Total number of outcomes = 6

    Rina throws a die and squares the number, so to get the number 25, the favourable outcome is 5.

    Favourable number of outcomes = 1

    ∴ Probability that Rina gets the square of the number as 25 
    fraction numerator Favourable space number space of space outcomes over denominator Total space number space of space outcomes end fraction space equals fraction numerator space 1 over denominator 6 end fraction
    As, 1/6>1/36, so Rina has better chance to get the number 25.

    Question 224
    CBSEENMA10009683

    Two different dice are tossed together. Find the probability:
    (i) of getting a doublet
    (ii) of getting a sum 10, of the numbers on the two dice.

    Solution

    The outcomes when two dice are thrown together are
    (1,1), (1,2), (1,3), (1,4), (1,5), (1,6)
    (2,1), (2,2), (2,3), (2,4), (2,5), (2,6)
    (3,1), (3,2), (3,3), (3,4), (3,5), (3,6)
    (4,1), (4,2), (4,3), (4,4), (4,5), (4,6)
    (5,1), (5,2), (5,3), (5,4), (5,5), (5,6)
    (6,1), (6,2), (6,3), (6,4), (6,5), (6,6)
    Total number of outcomes = 36
    n (s) = 36

    i) A = getting a doublet
    A = {(1,1), (2,2), (3,3),(4,4), (5,5), (6,6)}

    n(A) = 6
     P(A) = n(A)n(S) = 636 =16

    B = getting sum of numbers as 10.
    B = {(6, 4), (4, 6), (5, 5)}
    n(B) =3

     P (B) = n(B)n(S) = 336 = 112

    Question 225
    CBSEENMA10009685

    An integer is chosen at random between 1 and 100. Find the probability that it is :

    (i) divisible by 8

    (ii) not divisible by 8

    Solution

    An integer is chosen at random from 1 to 100
    Therefore n(S) = 100
    (i) Let A be the event that number chosen is divisible by 8
    ∴ A = { 8,16,24,32,40,48,56,64,72,80,88,96}
    ∴ n (A) = 12
    Now, P (that number is divisible by 8)

    P(A) = n(A)n(S) = 12100 = 650 = 325P (A) = 325

    (ii) Let ‘A’ be the event that number is not divisible by 8.

     P (A') = 1-P(A) = 1-325P(A') = 2225

    Question 226
    CBSEENMA10009715

    The probability of getting an even number, when a die is thrown once, is

    • 12

    • 13

    • 16

    • 56

    Solution

    A.

    12

    S = { 1, 2, 3, 4, 5, 6 }

    let event E be defined as 'getting an even number'.

    n(E) = { 2, 4, 6 }

      P(E) = Number of favourable outcomesNumber of possible outcomes =36                  = 12

    Question 227
    CBSEENMA10009716

    A box contains 90 discs, numbered from 1 to 90. If one disc is drawn at random from the box, the probability that it bears a prime-number less than 23,is

    • 790

    • 1090

    • 445

    • 989

    Solution

    C.

    445

    S = { 1, 2, 3,........90 }

    n(s) = 90

    The prime number less than 23 are 2, 3, 5, 7, 11, 13, 17 and 19

    Let event E be defined as 'getting a prime number less than 23'.

    n(E) = 8

     p(E) =Number of favourable outcomesNumber of possible outcomes = 890               =445

    Question 228
    CBSEENMA10009721

    A card is drawn at random from a well shuffled pack of 52 playing cards.Find the probability that the drawn card is neither a king nor a queen

    Solution

    Let  E be the event that the drwan card is neither a king nor a queen.

    Total number of possible outcomes = 52

    total number of kings  and queens = 4+4 = 8

    Therefore, there are 52 - 8 = 44 cards that are neither king nor queen.

    total number of favourable outcomes = 44

      Required probability = P(E) = Favourable outcomestotal number of outcomes = 4452                                                         =  1113

    Question 229
    CBSEENMA10009728

    A Group consists of 12 persons, of which 3 are extremely patient, other 6 are extremely honest and rest are extremely kind. A person from the group is selected at random. Assuming that each person is equally likely to be selected, find the probability of selecting a person who is

    (i)extremely patient

    (ii) extremely kind or honest. Which of the above values you prefer more?

    Solution

    The group consist of 12 persions.

     The number of possible outcomes = 12Let A denote event of selecting persons who are extremely patient Number of outcomes favourable to A is 3.Let be denote event of selecting persons who are extremely kind or honest.Number of persons who are extremely honest is 6.Number of persons who are extremely kind is 12 - (6 + 3) = 3 Number of outcomes favourable to B = 6+3 = 9.(i)  P(A) =  Number of outcomes favourable to ATotal number of possible outcomes = 312 = 14(ii ) P(B) =   Number of outcomes favourable to BTotal number of possible outcomes = 912 = 34Each of the three values, patience, honesty and kindness is important in one's life.

    Question 230
    CBSEENMA10009748

    The probability that a number selected at random from the numbers 1, 2, 3, ..., 15 isa multiple of 4, is

    • 415

    • 212

    • 15

    • 13

    Solution

    C.

    15

    The multiple of 4 between  1  and  15 are  4, 8, 12. the probability of getting amultiple of 4 = 315  = 15

    Question 231
    CBSEENMA10009751

    In a family of 3 children, the probability of having at least one boy is

    • 78

    • 18

    • 58

    • 34

    Solution

    A.

    78

    There are in all 23 = 8 combinations or outcomes for the gender of the 3 children

    The 8 combinations are as follows

    BBB, BBG, BGB, BGG, GBB, GBG, GGB, GGG

    Thus the probability of having at least one boy in a family is 78

     

    Question 232
    CBSEENMA10009755

    Two different dice are tossed together. Find the probability


    (i) That the number on each die is even.


    (ii) That the sum of numbers appearing on the two dice is 5.

    Solution

     The total number of outcomes when two dice are tossed together is 36.

     The sample space is as follows

    (i).  Favourable outcomes = { (2,2) (2,4) (2,6) (4,2) (4,4)                                                               

                                                (6,4) (6,2) (6,4) (6,6) } 

    Porbability that the number on each dice is even

      =Number of favourable outcomesTotal number of outcomes = 936  = 14

     

    (ii)  Favourable outcomes = { (1,4) (2,3) (3,2) (4,1) }

        Probability that the sum of the number appearing on the two dice is 5

         = Number of favourable outcomesTotal number of outcomes = 436 = 19                                                                    

     

    Question 233
    CBSEENMA10009774

    All the red face cards are removed from a pack of 52 playing cards. A card is drawn at random from the remaining cards, after reshuffling them. Find the probability that the drawn card is


    (i) of red colour


    (ii) a queen


    (iii) an ace


    (iv) a face card

    Solution

    (i) Face card are removed from a pack of 52 playing card = 6

    Total favourable outcomes = 52 - 6 = 46

    Number of all possible outcomes = 26 - 6 = 20

    PE = 2046 = 1023   

    (ii) Number of all possible outcomes  a queen = 2

    PE = 246 = 123

     

    (iii) Number of all possible outcomes an ace = 2

    PE = 246 = 123

     

    (iv) Number of all possible outcomes = 6

    PE = 646 = 323

     

    Question 234
    CBSEENMA10009779

    The probability of selecting a rotten apple randomly from a heap of 900 apples is 0.18. What is the number of rotten apples in the heap?

    Solution

    Let the total number of rotten apples in a heap = n

    Total number of apples in a heap = 900

    probability of selecting a rotten apple from a heap = 0.18

    Now.

    P(selecting a rotten apple) = Number of rotten applesTotal number of apples0.18 = n900n = 0.18 x 900n = 162Hence, the number of rotten apples is 162.

    Question 235
    CBSEENMA10009790

    A bag contains 15 white and some black balls. If the probability of drawing a black ball from the bag is thrice that of drawing a white ball, find the number of black balls in the bag.

    Solution

    Let the number of black balls in the bag be x.

    Number of white balls = 15

    Hence, total number of balls in the bag = x + 15

     

    Given,  P(black ball) = 3 x P(white ball)

     

     xx + 15  = 3  x  15x + 15xx + 15  =  45x + 15 x = 45

    Thus, the number of black balls in the bag is 45.

    Question 236
    CBSEENMA10009800

    Two different dice are thrown together. Find the probability that the numbers obtained have


    (i) even sum, and


    (ii) even product

    Solution

    Elementary events associated to the random experiment of throwing two dice are:

    (1,1),  (1,2),  (1,3),  (1,4),  (1,5),  (1,6),

    (2,1),  (2,2),  (2,3),  (2,4),  (2,5),  (2,6),

    (3,1),  (3,2),  (3,3),  (3,4),  (3,5),  (3,6),

    (4,1),  (4,2),  (4,3),  (4,4),  (4,5),  (4,6),

    (5,1),  (5,2),  (5,3),  (5,4),  (5,5),  (5,6),

    (6,1),  (6,2),  (6,3),  (6,4),  (6,5),  (6,6),

     

    Total number of elementary events = 6 x 6 = 36.

     

    (i) Let A be the event of getting an even number as the sum.

    i.e.  2, 4, 6, 8, 10, 12.

    Elementary events favourable to event A are:

    (1,1),  (1,3),  (1,5),  (2,2),  (2,4),  (2,6),

    (3,1),  (3,3),  (3,5),  (4,2),  (4,4),  (4,6),

    (5,1),  (5,3),  (5,5),  (6,2),  (6,4),  (6,6).

     Total number of favourable events = 18.

    Hence,  required probability =  1836 = 12

     

    (ii) Let B be the event of getting an even number as the product.

    i.e.  2, 4,  6,  8,  10,  12,  16,  18,  20,  24,  30,  36.

     

    Elementary events favourable to event B are:

    (1,2),  (1,4),  (1,6),  (2,1),  (2,2),  (2,3),

    (2,4),  (2,5),  (2,6),  (3,2),  (3,4),  (3,6),

    (4,1),  (4,2),  (4,3),  (4,4),  (4,5),  (4,6),

    (5,2),  (5,4),  (5,6),  (6,1),  (6,2),  (6,3),

    (6,4),  (6,5),  (6,6).

     

     Total number of favourable events = 27.

    Hence, required probability =  2736 = 34.

    Question 237
    CBSEENMA10009812

    Two dice are thrown together. The probability of getting the same number on both dice is:

    • 12

    • 13

    • 16

    • 112

    Solution

    C.

    16

    When two dice are thrown together, the total number of outcomes is 36.

    favourable outcomes = { (1,1),  (2,2),  (3,3),  (4,4),  (5,5),  (6,6) }

     

     Required probability = Number of favourable outcomesTotal number of outcomes = 636 = 16

    Question 238
    CBSEENMA10009818

    A number is selected at random from first 50 natural numbers. Find the probability that it is a multiple of 3 and 4.

    Solution

    Total mumber of outcomes is 50.

     

    Favourable outcomes = { 12, 24, 36, 48 }

     

    Required probability = Number of favourable outcomestotal number of outcomes                                    = 450                                                                                      = 225

    Question 239
    CBSEENMA10009834

    A card is drawn from a well shuffled deck of 52 cards. Find the probability of getting

    (i) a king of red colour

    (ii) a face card

    (iii) the queen of diamonds.

    Solution

    Total number of outcomes = 52

    (i) Probability of getting  a red king

    Here the number of favourable outcomes = 2

     

    Probability = number of favourable outcomesTotal number of outcomes                     = 252 = 126

    (ii) probability of getting a face card

    Total number of face cards = 12

     

    Probability = number of favourable  outcomesTotal number of outcomes                     = 1252 = 313

     

    (iii) Probability of queen of diamonds

    Number of queens of diamond = 1

     

    Probability = Number of favourable outcomestotal number of outcomes                    = 152